Используя график зависимости скорости движения тела от времени определите ускорение тела: Используя график зависимости скорости движения тела от времени, определите его ускорение.

Содержание

1. График зависимости скорости от времени при прямолинейном движении с постоянным ускорением

Самое простое из всех неравномерных движений — это прямолинейное движение с постоянным ускорением.

 

При движении с постоянным ускорением (a→=const→) скорость тела линейно зависит от времени:

 

v→=v→o+a→t.

 

В проекциях на ось \(Ox\) данные равенства имеют вид:

 

ax=const;

 

vx=vox+axt.

 

Построим графики зависимостей axt и vxt для случаев ax>0 и ax<0.

Примем vox>0.

 

Поскольку в обоих случаях ax=const, то графиком зависимости axt ускорения от времени в обоих случаях будет прямая, параллельная оси времени.

Только при ax>0 данная прямая будет лежать в верхней полуплоскости (рис. \(1\)), а при ax<0 — в нижней (рис. \(2\)).

 

Рис. \(1\)

 

Рис. \(2\)

 

Графиком зависимости скорости движения тела от времени vxt является прямая, пересекающая ось скорости в точке v0 и образующая с положительным направлением оси времени острый угол при ax>0 (рис. \(3\)) и тупой угол при ax<0 (рис. \(4\)).

 

Рис. \(3\)

 

Рис. \(4\)

 

График на рисунке \(3\) описывает возрастание проекции скорости vx. При этом модуль скорости тела также растёт. Данный график соответствует равноускоренному движению тела.

 

График на рисунке \(4\) показывает, что проекция vx скорости тела вначале положительна.

Она уменьшается и в момент времени t=tп становится равной нулю.

В этот момент тело достигает точки поворота, в которой направление скорости тела меняется на противоположное, и при t>tп проекция скорости становится отрицательной.

 

Из последнего графика также видно, что до момента поворота модуль скорости уменьшался — тело двигалось равнозамедленно.

При t>tп модуль скорости растёт — тело движется равноускоренно.

Для любого равнопеременного прямолинейного движения площадь фигуры между графиком vx и осью времени \(t\) численно равна проекции перемещения Δrx.

Рис. \(5\)

 

Согласно данному правилу, проекция перемещения Δrx при равнопеременном движении определяется площадью трапеции \(ABCD\) (рис. \(5\)). Эта площадь равна полусумме оснований трапеции, умноженной на её высоту:

  

S=AB+DC2⋅AD.

  

В результате:

  

Δrx=vox&plus;vx2⋅Δt.

  

Из данной формулы получим формулу для среднего значения проекции скорости:

  

vxср=ΔrxΔt=vox&plus;vx2.

  

При движении с постоянным ускорением данное отношение выполняется не только для проекций, но и для векторов скорости:

  

vcp→=vo→&plus;v→2.

Средняя скорость движения с постоянным ускорением равна полусумме начальной и конечной скоростей.

Всего вопросов: 20

Вопрос 1. Изображен график скорости движения мотоцикла от времени. Чему равна скорость мотоцикла в момент времени t=5c?

Вопрос 2. На рисунке изображен график зависимости скорости прямолинейного движения тела от времени. Чему равно ускорение тела?

Вопрос 3. На рисунке изображен график зависимости скорости прямолинейного движения тела от времени. Чему равно ускорение тела?

Вопрос 4. На рисунке изображена зависимость скорости движения тела от времени. На каком из участков тело движется равноускоренно?

Вопрос 5. На рисунке представлен график зависимости скорости движения тела от времени. На каком из участков тело движется равноускоренно?

Вопрос 6. На рисунке представлен график зависимости скорости движения тела от времени. На каком из участков тело движется равноускоренно?

Вопрос 7. На рисунке изображен график зависимости скорости движения тела от времени. Используя данные графика, запишите уравнение зависимости скорости от времени движения тела.

Вопрос 8. Проекция скорости тела изменяется с течением времени так, как показано на рисунке. Какое из нижеприведенных уравнений соответствует зависимости координаты этого тела от времени? (В момент начала наблюдения тело находилось на расстоянии двух метров левее начала координат)

Вопрос 9. Проекция скорости тела изменяется с течением времени так, как показано на рисунке. Какое из нижеприведенных уравнений, соответствует зависимости координаты этого тела от времени? (Учитывая, что в момент начала наблюдения рассматриваемая точка находилась на расстоянии 5 м левее начала координат)

Вопрос 10. По графику зависимости модуля скорости от времени, представленному на рисунке, определите перемещение тела за 2 с.

Вопрос 11. Используя информацию, приведенную на рисунке, определить проекцию перемещения тела через 14 с после начала движения.

Вопрос 12. Используя информацию, приведенную на рисунке, определить путь пройденный телом за девять секунд.

Вопрос 13. Автомобиль начинает двигаться равноускоренно и вдруг тормозит. Какой вид графика соответствует зависимости ускорения автомобиля от времени?

Вопрос 14. На рисунке 1 изображен график зависимости ускорения от времени движения тела. Как зависит скорость движения этого тела от времени (рисунок 2), если начальная скорость равна нулю?

Вопрос 15. На рисунке приведен график зависимости проекции скорости тела от времени. Определить в какой момент времени тело остановилось.

Вопрос 16. На рисунке представлен график зависимости проекции скорости от проекции перемещения. Определить ускорение этого тела.

Вопрос 17. На рисунке представлен график зависимости проекции скорости двух тел от времени. Определите скорость первого тела через три секунды после начала движения.

Вопрос 18. Определить, в каком соотношении между собой находятся проекции перемещения тел, графики зависимости проекций скоростей от времени которых, показаны на рисунке, в момент времени, когда скорости тел одинаковы?

Вопрос 19. На рисунке приведен график зависимости проекции скорости трех тел от времени. В каком из нижеприведенных соотношений находятся значения модулей ускорений и перемещений этих тел в момент времени 10 с?

Вопрос 20. Тело, имеющее начальную скорость 2 м/с, направленную против выбранной оси координат, двигается с ускорением, график зависимости проекции которого от времени приведен на рисунке. Какой из нижеприведенных графиков соответствует зависимости проекции скорости этого тела от времени для промежутка времени (0, 8) с?


Разбор тренировочного теста

Разбор тренировочного теста интернет-олимпиады
по физике 2008/2009 года

11 класс. Кинематика

 

Вопрос № 1

По графику, представленному на рисунке, определите скорость
движения велосипедиста через три секунды после начала движения.

 

Решение.

На рисунке представлен график зависимости пути от времени.
График представляет собой прямую линию, значит, велосипедист двигался
равномерно. Определим по графику величину пути, пройденного велосипедистом за
фиксированный отрезок времени. Например, за 3 с велосипедист прошел 9 м.
Скорость велосипедиста V = L / t
= 9/3 = 3 м/с.

 

Вопрос № 2

Пешеход и велосипедист одновременно начали движение
навстречу. Их скорости равны V1 = 6 км/ч и V2 = 30 км/ч, соответственно.
Определите время движения до встречи, если начальное расстояние между ними L = 700 м.

 

Решение.

Определим скорость велосипедиста в системе отсчета пешехода V12
= V1 + V2 = 6 + 30 = 36 км/ч = 10 м/с. Итак, пешеход и
велосипедист сближаются со скоростью 10 м/с, тогда их время движения до встречи
t = L / V12 = 700/10 = 70 с.

 

Вопрос № 3

Автомобиль двигался со скоростью 15 м/с
в течение 5 с. Какой путь он проехал за это время?

 

Решение.

Автомобиль двигался равномерно, поэтому пройденный путь L = V∙t = 15∙5 = 75 м.

 

Вопрос № 4

Брошенный вертикально вверх мяч возвращается в исходное
положение. На рисунке представлен график его скорости от времени. В какой
момент времени мяч достиг максимальной высоты?

 

Решение.

В момент, когда мяч достиг максимальной высоты, его скорость
равна нулю. По графику, представленному на рисунке определяем, что скорость
мяча равна нулю в момент времени t = 2 с.

 

Вопрос № 5

Какие из перечисленных выше величин векторные ? (Отметьте все векторные величины)

 

Решение.

Из перечисленных величин векторными являются скорость,
ускорение и перемещение. Путь — величина скалярная.

 

Вопрос № 6

Спортсмен пробежал дистанцию 400 м по дорожке стадиона и
возвратился к месту старта. Определите путь L, пройденный спортсменом, и модуль
его перемещения S.

 

Решение.

Пройденный спортсменом путь L = 400 м. Модуль перемещения S
= 0, так как спортсмен вернулся в точку, из которой он начал движение.

 

Вопрос № 7

Скорость тела, движущегося прямолинейно и равноускоренно,
изменилась при перемещении из точки 1 в точку 2 так, как показано на рисунке.
Какое направление имеет вектор ускорения на этом участке пути?

 

Решение.

Из рисунка видно, что модуль скорости тела при перемещении
уменьшается, значит, вектор ускорения направлен навстречу движению, то есть налево.

 

Вопрос № 8

По графику зависимости модуля скорости от времени определите
ускорение прямолинейно движущегося тела в момент времени t
= 2 с.

 

Решение.

По графику определим изменение скорости тела за
фиксированный момент времени. Например, за первые две секунды скорость тела
изменилась на 6 м/с (с V0
= 3 м/с до Vt = 9 м/с). Ускорение a = (Vt – V0) / t
= 6/2 = 3 м/с2.

 

Вопрос № 9

При равноускоренном движении автомобиля в течение пяти
секунд его скорость увеличилась от 10 до 15 м/с. Чему равен модуль ускорения автомобиля?

 

Решение.

Ускорение автомобиля a = (Vt – V0) / t = (15 – 10)/5 = 5/5 = 1 м/с2.

 

 

Вопрос № 10

Автомобиль стартует с места с постоянным ускорением а = 1
м/с2. Какой путь проходит автомобиль за
первые десять секунд движения?

 

Решение.

Автомобиль движется равноускоренно без начальной скорости —
пройденный путь L = a∙t2/2 = 1∙102/2
= 50 м.

 

Вопрос № 11

Плот равномерно плывет по реке со скоростью 3 км/ч. Сплавщик
движется поперек плота со скоростью 4 км/ч. Какова скорость сплавщика в системе отсчета, связанной с
берегом?

 

Решение.

Скорость сплавщика в в
системе отсчета, связанной с берегом

 

Вопрос № 12

Вертолет поднимается вертикально вверх c
постоянной скоростью. Какова траектория движения точки на конце лопасти винта
вертолета в системе отсчета, связанной с корпусом вертолета?

 

Решение.

Представьте себе, что вы находитесь в кабине вертолета, то
есть вы неподвижны относительно корпуса вертолета. В этом случае вы можете
видеть, что любая точка винта вертолета описывает окружность.

 

Вопрос № 13

Тело движется вдоль оси Х по закону, представленному на
рисунке, где х — координата в метрах, t — время в секундах. Определите модуль ускорения тела.

 

Решение.

Уравнение зависимости координаты от времени при
прямолинейном равноускоренном движении в общем виде имеет вид Х(t) = X0 + V∙t + aх∙t2/2, где X0 — начальная
координата, а V
и aх— проекции начальной скорости и ускорения на ось Х.

Приравнивая члены, в которые входит t2, получим aх∙t2/2 = –4,5∙t2. Откуда проекция ускорения aх
= –9 м/с2, а модуль ускорения a =
9 м/с2.

 

Вопрос № 14

На рисунке представлены графики зависимости модуля скорости
от времени для четырех тел. Какое из этих тел (или
какие тела) прошли наибольший путь?

 

Решение.

На рисунке показаны графики зависимости скорости движущихся
тел от времени. Как известно, пройденный телом путь представляет собой площадь,
лежащую под графиком скорости. Из рисунка видно, что фигура максимальной
площади лежит под графиком, для тела 4. Значит, за промежуток времени от 0 до t0
тело 4 прошло наибольший путь.

 

Вопрос № 15

Тело движется прямолинейно. На рисунке представлен график
скорости тела от времени. На каком промежутке (каких промежутках) времени
проекция ускорения отрицательна?

 

Решение.

Проанализируем график:

1.      на
промежутке времени от 0 до 1с скорость тела постоянна, поэтому ах =
0;

2.      на
промежутке времени от 1с до 2с скорость тела уменьшается, поэтому проекция
ускорения ах < 0;

3.      на
промежутке времени от 2с до 3с тело покоится, поэтому ах = 0;

4.      на
промежутке времени от 3с до 4с скорость тела увеличивается, поэтому проекция
ускорения ах > 0.

Итак, проекция ускорения отрицательна на промежутке времени
от 1с до 2с.

 

Вопрос № 16

Двигавшийся с начальной скоростью 20 м/с автомобиль
разгоняется с постоянным ускорением а = 2 м/с2
в течение 5 с. Какой путь он проехал за это время?

 

Решение.

Для расчета пути можно воспользоваться формулой L = V0∙t + a∙t2/2
= 20∙5 + 2∙52/2 = 125 м.

 

Итоговая контрольная работа по физике 10 класс

ЭКЗАМЕНАЦИОННАЯ КОНТРОЛЬНАЯ РАБОТА ПО ФИЗИКЕ

В 10 КЛАССЕ

ВАРИАНТ 1

ЧАСТЬ А: Выберите один верный ответ.

  1. Используя график зависимости скорости движения тела от времени, определите скорость тела в конце 7-ой секунды, считая, что характер движения тела не изменится.

1) 8 м/с 2) 11 м/с 3) 16 м/с

4) 18 м/с

  1. Зависимость пути от времени для прямолинейно движущегося тела имеет вид: S(t) = 2t + t2, где все величины выражены в СИ. Ускорение тела равно

1) 1 м/с2 2) 2 м/с2 3) 3 м/с2 4) 6 м/с2

  1. Тело упало с некоторой высоты с нулевой начальной скоростью и при ударе о землю имело скорость 40 м/с. Чему равно время падения? Сопротивлением воздуха пренебречь.

1) 0,25 с 2) 4 с 3) 40 с 4) 400с

  1. Материальная точка движется по окружности с постоянной скоростью. Как изменится центростремительное ускорение точки, если скорость увеличить в 2 раза и радиус окружности увеличить в два раза?

1) уменьшится в 2 раза 2) увеличится в 2 раза

3) увеличится в 4 раза 4) уменьшится в 8 раз

  1. Человек вез ребенка на санках по горизонтальной дороге. Затем на санки сел второй такой же ребенок, но человек продолжал движение с той же постоянной скоростью. Как изменилась сила трения при этом?

1) не изменилась 2) увеличилась в 2 раза

3) уменьшилась в 4 раза 4) увеличилась на 50 %

  1. По наклонной плоскости скользит брусок. Какой вектор, изображенный на рисунке, является лишним или неправильным?

1) 2) 3) 4)

  1. Идеальный газ сначала нагревался при постоянном давлении, потом его давление уменьшилось при постоянном объеме, затем при постоянной температуре давление газа увеличилось до первоначального значения. Какой из графиков в координатах р-Т соответствует этим изменениям состояния газа?

  1. На V-T – диаграмме представлен процесс изменения состояния идеального одноатомного газа. При переходе из состояния 1 в состояние 2 газ отдал 80 кДж теплоты. Внутренняя энергия этого газа

1) увеличилась на 80 кДж 2) уменьшилась на 80 кДж

3) увеличилась на 40 кДж 4) уменьшилась на 40 кДж

  1. Как изменится электроемкость плоского воздушного конденсатора при уменьшении расстояния между его пластинами в 2 раза и введении между пластинами диэлектрика с диэлектрической проницаемостью, равной 4?

1) увеличится в 8 раз 2) увеличится в 2 раза

3) уменьшится в 2 раз 4) не изменится

  1. ЭДС источника равна 8 В, внешнее сопротивление 3 Ом, внутреннее сопротивление 1 Ом. Сила тока в полной цепи равна

1) 32 А 2) 25 А 3) 2 А 4) 0,5 А

ЧАСТЬ В. Используя условие задачи, установите соответствие величин из левого столбца с их соотношениями из правого столбца.

  1. На аэрозольном баллончике написано: «… беречь от попадания прямых солнечных лучей и нагрева выше 50ºС…». Это требование обусловлено тем, что при нагревании…

А. масса газа

Б. температура газа 1) увеличивается

В. давление газа 2) уменьшается

Г. объем газа 3) не изменяется

Решите задачи:

  1. Подвешенное к тросу тело массой 10 кг поднимается вертикально. С каким ускорением движется тело, если трос жесткостью 59 кН/м удлинится на 2 мм? Какова сила упругости, возникающая в тросе?

  1. Вода падает с высоты 1200 м. На сколько повысится температура воды, если на ее нагревание затрачивается 60 % работы силы тяжести?

ЧАСТЬ С. Решите задачи.

  1. Автомобиль, идущий со скоростью 36 км/ч, начинает двигаться с ускорением 0,2 м/с2. Какой путь пройдет автомобиль за десятую секунду от начала движения?

  1. Температура однородного медного цилиндрического проводника длиной 10 м в течение 57 с повысилась на 10 К. Определить напряжение, которое было приложено к проводнику в это время. Изменением сопротивления проводника и рассеянием тепла при его нагревании пренебречь.

ЭКЗАМЕНАЦИОННАЯ КОНТРОЛЬНАЯ РАБОТА ПО ФИЗИКЕ

В 10 КЛАССЕ

ВАРИАНТ 2

ЧАСТЬ А: Выберите один верный ответ.

  1. Используя график зависимости скорости движения тела от времени, определите скорость тела в конце 8-ой секунды, считая, что характер движения тела не изменится.

1) 35 м/с 2) 30 м/с 3) 25 м/с

4) 21 м/с

  1. Зависимость пути от времени для прямолинейно движущегося тела имеет вид: S(t) = 3t — t2, где все величины выражены в СИ. Ускорение тела равно

1) 1 м/с2 2) 2 м/с2 3) 3 м/с2 4) 6 м/с2

  1. Камень брошен вертикально вверх со скоростью 50 м/с. Через сколько секунд его скорость будет 30 м/с и направлена вертикально вверх?

1) 2 с 2) 6 с 3) 8 с 4) 10 с

  1. Материальная точка движется по окружности с постоянной скоростью. Как изменится центростремительное ускорение точки, если скорость уменьшить в 2 раза и радиус окружности в 2 раза увеличить?

1) уменьшится в 2 раза 2) увеличится в 2 раза

3) увеличится в 4 раза 4) уменьшится в 8 раз

  1. Человек вез двух одинаковых детей на санках по горизонтальной дороге. Затем с санок встал один ребенок, но человек продолжал движение с той же постоянной скоростью. Как изменилась сила трения при этом?

1) не изменилась 2) уменьшилась в 2 раза

3) увеличилась в 2 раза 4) увеличилась на 50 %

  1. По наклонной плоскости равномерно вверх перемещается брусок. Какой вектор, изображенный на рисунке, является лишним или неправильным?

1) 2) 3) 4)

  1. Идеальный газ сначала охлаждался при постоянном давлении, потом его давление уменьшилось при постоянном объеме, затем при постоянной температуре объем газа увеличился до первоначального значения. Какой из графиков в координатах р-V соответствует этим изменениям состояния газа?

  1. На Т-Р – диаграмме показан процесс изменения состояния идеального газа неизменной массы. Газ совершил работу, равную 5 кДж. Количество теплоты, полученное газом, равно

1) 0 кДж 2) 3 кДж

3) 3,5 кДж 4) 5 кДж

  1. Как изменится электроемкость плоского воздушного конденсатора при увеличении расстояния между его пластинами в 2 раза и введении между пластинами диэлектрика с диэлектрической проницаемостью, равной 2?

1) увеличится в 4 раза 2) увеличится в 2 раза

3) уменьшится в 2 раз 4) не изменится

  1. Сила тока в полной цепи 8 А, внешнее сопротивление 4 Ом, внутреннее сопротивление 1 Ом. ЭДС источника равна

1) 40 В 2) 33 В 3) 3 В 4) 0,5 В

ЧАСТЬ В. Используя условие задачи, установите соответствие величин из левого столбца с их соотношениями из правого столбца.

  1. На аэрозольном баллончике написано: «… беречь от попадания прямых солнечных лучей и нагрева выше 50ºС…». Это требование обусловлено тем, что при нагревании…

А. масса молекулы газа

Б. количество молекул 1) увеличивается

В. скорость молекул газа 2) уменьшается

Г. давление газа 3) не изменяется

Решите задачи:

  1. Автобус массой 15 т трогается с места с ускорением 0,7 м/с2. Какая сила трения действует на автобус, если сила тяги двигателя равна 15 кН? Ответ выразить в кН. Чему равен коэффициент трения?

  1. Снаряд, летевший со скоростью 200 м/с ударяется в земляную насыпь и застревает в ней. На сколько градусов повысится температура снаряда, если на его нагревание пошло 60 % кинетической энергии? Удельная теплоемкость вещества снаряда 400 Дж/(кг. º С).

ЧАСТЬ С. Решите задачи.

  1. Спортсмен пробежал расстояние 100 м за 10 с, из которых он 2 с потратил на разгон, а остальное время двигался равномерно. Чему равна скорость равномерного движения?

  1. К однородному медному цилиндрическому проводнику длиной 10 м приложили разность потенциалов 1 В. Определите промежуток времени, в течение которого температура проводника повысится на 10 К. Изменением сопротивления проводника и рассеиванием тепла при его нагревании пренебречь.

ИЗД Механика поступательного и вращательного движения. 16.1. Используя график зависимости скорости движения тела от времени, определите скорость тела в конце пятой секунды

Вариант 16

16.1. Используя график зависимости скорости движения тела от времени, определите скорость тела в конце пятой секунды, считая, что характер движения не изменился. Определите координату x в момент времени t = 3 c, если в начальный момент времени t0 = 0 с координата x0 = 2 м.

16.2. Грузы массами m1 = 4 кг и m2 = 5 кг связаны между собой нитью, перекинутой через неподвижный блок. Первый груз расположен на горизонтальном столе, второй висит на вертикальной части нити. Определите ускорение грузов a и силу натяжения нити T при условии, что коэффициент трения груза о стол μ = 0,1. Нить считать невесомой и нерастяжимой, массой блока пренебречь.

16.3. Проекция px импульса материальной точки, движущейся прямолинейно вдоль оси Х, изменяется со временем так, как показано на рис. 1. Какой номер графика на рис. 2 соответствует зависимости Fx(t) – проекции силы, действующей на точку и вызывающей это изменение px? Ответ обосновать.

16.4. Два шарика, массы которых m = 0,1 кг и М = 0,2 кг, висят, соприкасаясь, на вертикальных нитях длиной l = 1,5 м (см. рисунок). Левый шарик отклоняют на угол 90° и отпускают без начальной скорости. Какое количество теплоты выделится в результате абсолютно неупругого удара шариков?

16.5. На рисунке приведен график параболической зависимости модуля углового перемещения абсолютно твердого тела от времени. Начальная угловая скорость равна нулю. За четыре секунды тело повернется на угол …рад.

16.6. Стержень вращается вокруг оси, проходящей через его один из его концов согласно уравнению φ = At+Bt3, где A=2 рад/с; B = 0,2 рад/с3. Найдите закон изменения момента сил M, действующих на стержень. Определите угловую скорость и момент сил M в момент времени t = 2 с, если момент инерции стержня J = 0,048 кг∙м2.

16.7. На рисунке представлен график зависимости кинетической энергии вращающегося Wвр тела, момент инерции которого равен J = 2 кг·м2 , от угла поворота φ. Чему равен вращающий момент сил, действующий на тело, в тот момент времени, когда угол поворота станет равным

φ = 3 рад.

16.8. Мальчик катит обруч по горизонтальной поверхности со скоростью v=7.2 км/ч., Найдите высоту (в метрах), на которую может вкатиться обруч в горку за счет своей кинетической энергии, если пренебречь силой трения качения.

Контрольная работа № 1

Контрольная работа №
1  (9 класс)

«Кинематика
прямолинейного движения»

В – 1

 

1. Какие из перечисленных величин являются скалярными?

А. Путь;          Б.
Перемещение;       В. Скорость;              Г. Ускорение.

 

2. Какое из уравнений описывает равноускоренное движение?

А.       Б.                 В.             Г.  

3. Ускорение автомобиля, начавшего движение, равно 0,5 м/с2. Какой путь пройдет автомобиль
за промежуток времени 4 секунды, двигаясь с этим ускорением?

 

4. Движение тела задано уравнением  (м). Какой будет его скорость через промежуток времени 5 с
после начала отсчета времени?

 

5. По заданному графику зависимости скорости от времени напишите
уравнение движения. Начальная координата тела равна нулю.

6. Вагон движется со скоростью 72 км/ч. Дождевые капли в
безветренную погоду оставляют след на его окне под углом 60о к
вертикали. Найдите скорость падения капель.

 

7. Автомобиль доставил груз из пункта А в пункт В,
перемещаясь в среднем со скоростью 40 км/ч. Возвращаясь обратно, автомобиль
двигался со средней скоростью 60 км/ч. Чему равна средняя скорость его движения
на всем пути?

 

8. Используя график скорости , постройте графики перемещения   и ускорения   и объясните характер
движения тела на различных участках .

 

9. Тело, двигаясь равноускоренно, в течение пятой секунды от
начала движения прошло 45м. С каким ускорением двигалось тело? Какова его
скорость в конце пятой секунды? Какой путь прошло тело за первую секунду?

 


Контрольная работа №
1  (9 класс)

«Кинематика
прямолинейного движения»

В – 2

 

1. Какое из уравнений описывает равномерное движение?

А.       Б.          В.       Г. 

2. Что называется перемещением?

А. Путь, который проходит тело;

Б. Вектор, соединяющий начальную и конечную точки траектории
движения тела за данный промежуток времени;

В. Длина траектории движения;

Г. Путь, который проходит тело за единицу времени.

 

3. Поезд отходит от станции с ускорением 1 м/с2. Определите промежуток времени,
за который поезд пройдет путь 8∙102 м.

 

4. Движение тела задано уравнением (м). Определите путь, пройденный за промежуток времени
10 с.

 

5. По графику зависимости модуля скорости от времени
определите ускорение и запишите уравнение движения. Начальная координата тела
равна 6 м.

 

6. Два поезда идут навстречу друг другу со скоростями 36 км/ч и 54 км/ч. Пассажир,
находящийся в поезде, замечает, что второй поезд проходит мимо него в течение 6
с. Найдите длину второго поезда.

 

7. Используя график зависимости проекции скорости от
времени, постройте графики зависимости ускорения, и пути от времени.

8. Поезд движется на подъеме со скоростью 10 м/с и
затем на спуске со скоростью 25 м/с. Какова средняя скорость поезда на всем
пути, если длина спуска в 2 раза больше длины подъема?

 

9. Реактивный самолет летит со скоростью 720 км/ч. С
некоторого момента самолет движется с ускорением в течение 10 с и в последнюю
секунду проходит путь 295 м.
Определите ускорение и конечную скорость.


Контрольная работа №
1  (9 класс)

«Кинематика
прямолинейного движения»

В – 3

 

1.Какие из перечисленных величин являются векторными?

А. Скорость;  Б.
Координата;         В. Пройденный путь;           Г. Время.

 

2. В каком из следующих случаев движение тела можно
рассматривать как движение материальной точки?

А. Вращение детали, обрабатываемой на токарном станке;

Б. Движение поезда по мосту;

В. Движение фигуриста на льду;

Г. Полет самолета, совершающего рейс Минск – Москва.

 

3. Поезд, движущийся после начала торможения с ускорением
0,4 м/с2, через 25 секунд остановился. Найдите скорость поезда в
начале торможения.

 

4. Движение тела задано уравнением  (м). Какой будет его скорость через 2 с после начала
отсчета времени?

 

5. По заданному графику зависимости скорости от времени
напишите уравнение движения. Начальная координата тела равна 5 м.

 

6. По прямолинейной дороге в одном направлении едут
велосипедист со скоростью 36 км/ч
и мотоциклист со скоростью 72 км/ч.
В начальный момент времени расстояние между ними было 250 м. Через какой промежуток времени
мотоциклист догонит велосипедиста?

 

7. Первую половину пути велосипедист ехал со скоростью 24 км/ч, а вторую – со скоростью . Чему равна эта скорость, если средняя скорость равна 12 км/ч.

 

8. На рисунке представлен график зависимости проекции
скорости на ось OX от времени. Начертите соответствующие этому графику
зависимости ускорения и перемещения от времени.

9. Тело при равноускоренном движении проходит за первые
4 с путь, равный 24 м.
Определить модуль начальной скорости тела, если за следующие 4 с тело
проходит расстояние 64 м.


Контрольная работа №
1  (9 класс)

«Кинематика
прямолинейного движения»

В – 4

1. Основная задача кинематики …

А. … установить причины движения тел;

Б. … изучить условия равновесия тел;

В. … определить положение тел в пространстве в любой момент
времени.

Г. … определить скорость движения.

 

2. Какой из представленных графиков выражает зависимость
ускорения материальной точки от времени при равноускоренном движении?

3. Поезд двигался со скоростью 72 км/ч. Найдите промежуток времени
∆t торможения, если известно, что
тормозной путь равен 800 м.

4. Движение тела задано уравнением  (м). Какой будет его скорость через 2 с после начала
отсчета времени?

 

5. По заданному графику зависимости скорости от времени
напишите уравнение движения. Начальная координата тела равна 3 м.

6. Катер, переправляясь через реку, движется перпендикулярно
течению реки со скоростью 4 м/с в системе отсчета, связанной с водой. На
сколько метров будет снесен катер течением, если ширина реки 800 м, а скорость течения 1 м/с?

 

7. Африканский страус имеет максимальную скорость 80 км/ч и с этой скоростью он
пробегает ¾ пути, а оставшуюся
часть пути он бежит со скоростью 40
км/ч. Какова средняя скорость страуса?

 

8. На рисунке представлен график зависимости проекции
скорости на ось OX от времени. Начертите соответствующие этому графику
зависимости ускорения и перемещения от времени.

      

 

9. За пятую секунду
равнозамедленного движения точка проходит 5
см и останавливается. Какой путь проходит точка за
третью секунду этого движения?

 

 

 

 

Скорость. Ускорение. Равноускоренное прямолинейное движение – FIZI4KA

1. Реальное механическое движение — это движение с изменяющейся скоростью. Движение, скорость которого стечением времени изменяется, называют неравномерным движением.

При неравномерном движении координату тола уже нельзя определить но формуле ​\( x=x_0+v_xt \)​, так как значение скорости движения не является постоянным. Поэтому для характеристики быстроты изменения положения тела с течением времени при неравномерном движении вводят величину, называемую средней скоростью.

Средней скоростью ​\( \vec{v}_{ср} \)​ неравномерного движения называют физическую величину, равную отношению перемещении \( \vec{s} \) тела ко времени ​\( t \)​, за которое оно произошло: ​\( \vec{v}_{ср}=\frac{s}{t} \)​.

Записанная формула определяет среднюю скорость как векторную величину. В практических целях этой формулой можно воспользоваться для определения модуля средней скорости лишь в том случае, когда тело движется вдоль прямой в одну сторону. Если же нужно определить среднюю скорость движения автомобиля от Москвы до Санкт-Петербурга и обратно, чтобы рассчитать расход бензина, то эту формулу применить нельзя, поскольку перемещение в этом случае равно нулю и средняя скорость тоже равна нулю. Поэтому на практике при определении средней скорости пользуются величиной, равной отношению пути ​\( l \)​ ко времени ​\( t \)​, за которое этот путь пройден: \( v_{ср}=\frac{l}{t} \). Эта скорость обычно называется средней путевой скоростью.

2. Важно, что, зная среднюю скорость неравномерного движения на каком-либо участке траектории, нельзя определить положение тела на этой траектории в любой момент времени. Например, если средняя скорость движения автомобиля за 2 часа 50 км/ч, то мы не можем сказать, где он находился через 0,5 часа от начала движения, через 1 час, 1,5 часа и т.п., поскольку он мог первые полчаса двигаться со скоростью 80 км/ч, затем какое-то время стоять, а какое-то время ехать в пробке со скоростью 20 км/ч.

3. Двигаясь по траектории, тело проходит последовательно все её точки. В каждой точке траектории оно находится в определённые моменты времени и имеет какую-то скорость.

Мгновенной скоростью называют скорость тела в данный момент времени в данной точке траектории.

Предположим, некоторое тело совершает неравномерное прямолинейное движение (рис. 17), его скорость в точке О можно определить следующим образом: выделим на траектории участок AB, внутри которого находится точка О. Перемещение тела на этом участке — \( \vec{s}_1 \) совершено за время \( t_1 \). Средняя скорость движения на этом участке – \( \vec{v}_{ср.1}=\frac{s_1}{t_1} \). Уменьшим перемещение тела. Пусть оно равно \( \vec{s}_2 \), а время движения — ​\( t_2 \)​. Тогда средняя скорость за это время: \( \vec{v}_{ср.2}=\frac{s_2}{t_2} \). Еще уменьшим перемещение, средняя скорость на этом участке: \( \vec{v}_{ср.3}=\frac{s_3}{t_3} \).

При дальнейшем уменьшении перемещения и соответственно времени движения тела они станут такими маленькими, что прибор, например спидометр, перестанет фиксировать изменение скорости, и движение за этот малый промежуток времени можно считать равномерным. Средняя скорость на этом участке и есть мгновенная скорость тела в т.О.

Таким образом, мгновенной скоростью называют векторную физическую величину, равную отношению малого перемещения (​\( \Delta{\vec{s}} \)​) к малому промежутку времени \( \Delta{t} \), за которое это перемещение произошло: ​\( \vec{v}=\frac{\Delta{s}}{\Delta{t}} \)​.

4. Одним из видов неравномерного движения является равноускоренное движение. Равноускоренным движением называют движение, при котором скорость тела за любые равные промежутки времени изменяется на одно и то же значение.

Слова «любые равные промежутки времени» означают, что какие бы равные промежутки времени (2 с, 1 с, доли секунды и т.п.) мы ни взяли, скорость всегда будет изменяться одинаково. При этом её модуль может как увеличиваться, так и уменьшаться.

5. Характеристикой равноускоренного движения, помимо скорости и перемещения, является ускорение.

Пусть в начальный момент времени ​\( t_0=0 \) ​скорость тела равна ​\( \vec{v}_0 \)​. В некоторый момент времени ​\( t \)​ она стала равной \( \vec{v} \). Изменение скорости за промежуток времени ​\( t-t_0=t \)​ равно ​\( \vec{v}-\vec{v}_0 \)​ (рис.18). Изменение скорости за единицу времени равно: \( \frac{\vec{v}-\vec{v}_0}{t} \). Эта величина и есть ускорение тела, она характеризует быстроту изменения скорости \( \vec{a}=\frac{\vec{v}-\vec{v}_0}{t} \).

Ускорение тела при равноускоренном движении — векторная физическая величина, равная отношению изменения скорости тела к промежутку времени, за который это изменение произошло.

Единица ускорения ​\( [a]=[v]/[t] \); ​\( [a] \)​​ = 1 м/с/1 с = 1 м/с2. 1 м/с2 — это такое ускорение, при котором скорость тела изменяется за 1 с на 1 м/с.

Направление ускорения совпадает с направлением скорости движения, если модуль скорости увеличивается, ускорение направлено противоположно скорости движения, если модуль скорости уменьшается.

6. Преобразовав формулу ускорения, можно получить выражение для скорости тела при равноускоренном движении: \( \vec{v}=\vec{v}_0+\vec{a}t \). Если начальная скорость тела ​\( v_0=0 \)​, то \( \vec{v} = \vec{a}t \).

Чтобы определить значение скорости равноускоренного движения в любой момент времени, следует записать уравнение для проекции скорости на ось ОХ. Оно имеет вид: \( v_x = v_{0x} + a_xt \); если\( v_{0x}=0 \), то \( v_x = a_xt \).

7. Как видно из формулы скорости равноускоренного движения, она линейно зависит от времени. Графиком зависимости модуля скорости от времени является прямая, составляющая некоторый угол с осью абсцисс (осью времени). На рисунке 19 приведены графики зависимости модуля скорости от времени.

График 1 соответствует движению без начальной скорости с ускорением, направленным так же, как и скорость; график 2 — движению с начальной скоростью \( v_{02} \) и с ускорением, направленным так же, как и скорость; график 3 — движению с начальной скоростью \( v_{03} \) и с ускорением, направленным в сторону, противоположную направлению скорости.

8. На рисунке приведены графики зависимости проекции скорости равноускоренного движения от времени (рис. 20).

График 1 соответствует движению без начальной скорости с ускорением, направленным вдоль положительного направления оси X; график 2 — движению с начальной скоростью \( v_{02} \), с ускорением и скоростью, направленными вдоль положительного направления оси X; график 3 — движению с начальной скоростью \( v_{03} \) : до момента времени \( t_0 \) направление скорости совпадает с положительным направлением оси X, ускорение направлено в противоположную сторону. В момент времени \( t_0 \) скорость равна нулю, а затем и скорость, и ускорение направлены в сторону, противоположную положительному направлению оси X.

9. На рисунке 21 приведены графики зависимости проекции ускорения равноускоренного движения от времени.

График 1 соответствует движению, проекция ускорения которого положительна, график 2 — движению, проекция ускорения которого отрицательна.

10. Формулу перемещения тела при равноускоренном движении можно получить, используя график зависимости проекции скорости этого движения от времени (рис. 22).

Выделим на графике малый участок ​\( ab \)​ и опустим перпендикуляры из точек​ \( a \)​ и ​\( b \)​ на ось абсцисс. Если промежуток времени ​\( \Delta{t} \)​, соответствующий участку ​\( cd \)​ на оси абсцисс мал, то можно считать, что скорость в течение этого промежутка времени не изменяется и тело движется равномерно. В этом случае фигура ​\( cabd \)​ мало отличается от прямоугольника и её площадь численно равна проекции перемещения тела за время, соответствующее отрезку ​\( cd \)​.2_x=2a_xs_x \)​.

Полученная формула позволяет рассчитать тормозной путь транспортных средств, т.е. путь, который проезжает, например, автомобиль до полной остановки. При некотором ускорении движения, которое зависит от массы автомобиля и силы тяги двигателя, тормозной путь тем больше, чем больше начальная скорость автомобиля.

ПРИМЕРЫ ЗАДАНИЙ

Часть 1

1. Hа рисунке приведены графики зависимости пути и скорости тела от времени. Какой график соответствует равноускоренному движению?

2. Автомобиль, начав двигаться из состояния покоя но прямолинейной дороге, за 10 с приобрел скорость 20 м/с. Чему равно ускорение автомобиля?

1) 200 м/с2
2) 20 м/с2
3) 2 м/с2
4) 0,5 м/с2

3. На рисунках представлены графики зависимости координаты от времени для четырёх тел, движущихся вдоль оси ​\( Оx \)​. У какого из тел в момент времени ​\( t_1 \)​ скорость движения равна нулю?

4. На рисунке представлен график зависимости проекции ускорения от времени для тела, движущегося прямолинейно вдоль оси ​\( Оx \)​.

Равноускоренному движению соответствует участок

1) только ОА
2) только АВ
3) только ОА и ВС
4) только CD

5. При изучении равноускоренного движения измеряли путь, пройденный телом из состояния покоя за последовательные равные промежутки времени (за первую секунду, за вторую секунду и т.д.). Полученные данные приведены в таблице.

Чему равен путь, пройденный телом за третью секунду?

1) 4 м
2) 4,5 м
3) 5 м
4) 9 м

6. На рисунке представлены графики зависимости скорости движения от времени для четырёх тел. Тела движутся по прямой.

Для какого(-их) из тел — 1, 2, 3 или 4 — вектор ускорения направлен противоположно вектору скорости?

1) только 1
2) только 2
3) только 4
4) 3 и 4

7. Используя график зависимости скорости движения тела от времени, определите его ускорение.

1) 1 м/с2
2) -1 м/с2
3) 2 м/с2
4) -2 м/с2

8. При изучении равноускоренного движения измеряли скорость тела в определённые моменты времени. Полученные данные, приведены в таблице. Чему равна скорость тела в момент времени 3 с?

1) 0 м/с
2) 2 м/с
3) 4 м/с
4) 14 м/с

9. На рисунке приведены графики зависимости скорости движения четырёх тел от времени. Ускорение какого из тел равно -1,5 м/с?

1) 1
2) 2
3) 3
4) 4

10. Используя график зависимости скорости движения тела от времени, определите скорость тела в конце 30-й секунды. Считать, что характер движения тела не изменился.

1) 14 м/с
2) 20 м/с
3) 62 м/с
4) 69,5 м/с

11. Два тела движутся по оси ​\( Оx \)​. На рисунке представлены графики зависимости проекции скорости движения тел 1 и 2 от времени.

Используя данные графика, выберите из предложенного перечня два верных утверждения. Укажите их номера.

1) В промежутке времени ​\( t_3-t_5 \)​ тело 2 движется равноускоренно.
2) К моменту времени ​\( t_2 \)​ от начала движения тела прошли одинаковые пути.
3) В промежутке времени ​\( 0-t_3 \)​ тело 2 находится в покое.
4) В момент времени ​\( t_5 \)​ тело 1 останавливается.
5) В промежутке времени ​\( t_3-t_4 \)​ ускорение ​\( a_x \)​ тела 1 отрицательно.

12. На рисунке представлен график зависимости проекции скорости от времени для тела, движущегося вдоль оси Ох.

Используя данные графика, выберите из предложенного перечня два верных утверждения. Укажите их номера.

1) Участок ОА соответствует ускоренному движению тела.
2) Участок АВ соответствует состоянию покоя тела.
3) В момент времени ​\( t_1 \)​ тело имело максимальное по модулю ускорение.
4) Момент времени ​\( t_3 \)​ соответствует остановке тела.
5) В момент времени ​\( t_2 \)​ тело имело максимальное по модулю ускорение.2 \)​. В какой момент времени скорость движения равна нулю?

Ответы

Скорость. Ускорение. Равноускоренное прямолинейное движение

3.4 (67.7%) 122 votes

3.3 Среднее и мгновенное ускорение — University Physics Volume 1

Цели обучения

К концу этого раздела вы сможете:

  • Рассчитайте среднее ускорение между двумя точками времени.
  • Рассчитайте мгновенное ускорение с учетом функциональной формы скорости.
  • Объясните векторную природу мгновенного ускорения и скорости.
  • Объясните разницу между средним ускорением и мгновенным ускорением.
  • Найдите мгновенное ускорение в заданное время на графике зависимости скорости от времени.

Важность понимания ускорения охватывает наш повседневный опыт, а также обширные просторы космического пространства и крошечный мир субатомной физики. В повседневном разговоре до ускоряться, означает ускоряться; нажатие на педаль тормоза приводит к замедлению движения автомобиля. Мы, например, знакомы с ускорением нашей машины. Чем больше ускорение, тем больше изменение скорости за заданный промежуток времени.Ускорение широко наблюдается в экспериментальной физике. Например, в экспериментах с линейным ускорителем частиц субатомные частицы ускоряются до очень высоких скоростей в экспериментах по столкновению, которые сообщают нам информацию о структуре субатомного мира, а также о происхождении Вселенной. В космосе космические лучи — это субатомные частицы, которые были ускорены до очень высоких энергий в сверхновых (взрывающихся массивных звездах) и активных ядрах галактик. Важно понимать процессы, которые ускоряют космические лучи, потому что эти лучи содержат очень проникающее излучение, которое может, например, повредить электронику, установленную на космических кораблях.

Среднее ускорение

Формальное определение ускорения согласуется с этими только что описанными понятиями, но является более всеобъемлющим.

Среднее ускорение

Среднее ускорение — это скорость изменения скорости:

где

— среднее ускорение, v — скорость, t — время. (Полоса над и означает среднее ускорение .)

Поскольку ускорение — это скорость в метрах, разделенная на время в секундах, единицы измерения ускорения в системе СИ часто обозначаются сокращенно: м / с 2 — то есть метры в секунду в квадрате или метры в секунду в секунду. Это буквально означает, на сколько метров в секунду скорость меняется каждую секунду. Напомним, что скорость — это вектор, он имеет как величину, так и направление, что означает, что изменение скорости может быть изменением величины (или скорости), но это также может быть изменение направления.Например, если бегун, движущийся со скоростью 10 км / ч на восток, замедляется до остановки, меняет направление, продолжает свой бег со скоростью 10 км / ч на запад, его скорость изменилась в результате изменения направления, хотя величина скорости одинаковы в обоих направлениях. Таким образом, ускорение происходит, когда скорость изменяется по величине (увеличение или уменьшение скорости) или по направлению, или по обоим направлениям.

Ускорение как вектор

Ускорение — это вектор в том же направлении, что и , изменение скорости ,

.Поскольку скорость является вектором, она может изменяться по величине или по направлению, или по обоим направлениям. Следовательно, ускорение — это изменение скорости или направления, или и того, и другого.

Имейте в виду, что хотя ускорение происходит в направлении изменения скорости, оно не всегда в направлении движения. Когда объект замедляется, его ускорение противоположно направлению его движения. Хотя это обычно называется замедлением (рисунок), мы говорим, что поезд ускоряется в направлении, противоположном его направлению движения.

Рисунок 3.10. Поезд метро в Сан-Паулу, Бразилия, замедляет скорость, когда заходит на станцию. Он ускоряется в направлении, противоположном направлению его движения. (Источник: Юсуке Кавасаки)

Термин замедление может вызвать путаницу в нашем анализе, поскольку он не является вектором и не указывает на конкретное направление относительно системы координат, поэтому мы его не используем. Ускорение — это вектор, поэтому мы должны выбрать для него соответствующий знак в выбранной нами системе координат.В случае поезда на (Рисунок) ускорение составляет в отрицательном направлении в выбранной системе координат , поэтому мы говорим, что поезд испытывает отрицательное ускорение.

Если движущийся объект имеет скорость в положительном направлении относительно выбранной исходной точки и приобретает постоянное отрицательное ускорение, объект в конечном итоге останавливается и меняет направление на противоположное. Если мы подождем достаточно долго, объект пройдет через начало координат в противоположном направлении. Это проиллюстрировано на (Рисунок).

Рис. 3.11. Объект, движущийся с вектором скорости на восток при отрицательном ускорении, останавливается и меняет направление на противоположное. Через достаточно долгое время он проходит исходную точку в обратном направлении.

Пример

Расчет среднего ускорения: скакун покидает ворота

Скаковая лошадь, выходящая из ворот, ускоряется из состояния покоя до скорости 15,0 м / с на запад за 1,80 с. Какое у него среднее ускорение?

Рисунок 3.12 Скаковые лошади ускоряются из-за ворот. (кредит: Джон Салливан)

Стратегия

Сначала мы рисуем эскиз и назначаем систему координат задаче (рисунок). Это простая проблема, но всегда помогает ее визуализировать. Обратите внимание, что мы назначаем восток как положительный, а запад как отрицательный. Таким образом, в этом случае мы имеем отрицательную скорость.

Рисунок 3.13 Определите систему координат, данную информацию и то, что вы хотите определить.

Мы можем решить эту проблему, указав

на основе предоставленной информации, а затем вычисление среднего ускорения непосредственно из уравнения

.

Решение

Сначала определите известных:

(знак минус указывает направление на запад), Δ t = 1,80 с.

Во-вторых, найдите изменение скорости. Поскольку лошадь движется с нуля до –15,0 м / с, ее изменение скорости равно ее конечной скорости:

Наконец, подставьте известные значения (

) и найти неизвестное

:

Значение

Отрицательный знак ускорения указывает на то, что ускорение направлено на запад.Ускорение 8,33 м / с 2 на западе означает, что лошадь увеличивает свою скорость на 8,33 м / с на западе каждую секунду; то есть 8,33 метра в секунду в секунду, что мы записываем как 8,33 м / с 2 . Это действительно среднее ускорение, потому что поездка не гладкая. Позже мы увидим, что ускорение такой величины потребовало бы от всадника держаться с силой, почти равной его весу.

Проверьте свое понимание

Протонов в линейном ускорителе ускоряются из состояния покоя до

.

за 10 –4 с.Какое среднее ускорение протонов?

[показывать-ответ q = ”fs-id1168327875120 ″] Показать решение [/ показывать-ответ]

[скрытый-ответ a = ”fs-id1168327875120 ″]

Вставив знания, получим

[/ hidden-answer]

Мгновенное ускорение

Мгновенное ускорение a или ускорение в определенный момент времени получается с использованием того же процесса, который описан для мгновенной скорости.То есть мы вычисляем среднюю скорость между двумя моментами времени, разделенными

и пусть

приближаются к нулю. Результатом является производная функции скорости v ( t ), которая составляет мгновенное ускорение и математически выражается как

.

Таким образом, подобно тому, как скорость является производной функции положения, мгновенное ускорение является производной функции скорости.Мы можем показать это графически так же, как мгновенную скорость. На (Рисунок) мгновенное ускорение в момент времени t 0 — это наклон касательной к графику зависимости скорости от времени в момент времени t 0 . Видим, что среднее ускорение

приближается к мгновенному ускорению как

приближается к нулю. Также в части (а) рисунка мы видим, что скорость имеет максимум, когда ее наклон равен нулю.Это время соответствует нулю функции ускорения. В части (b) показано мгновенное ускорение при минимальной скорости, которая также равна нулю, поскольку наклон кривой там также равен нулю. Таким образом, для данной функции скорости нули функции ускорения дают либо минимальную, либо максимальную скорость.

Рис. 3.14. На графике зависимости скорости от времени мгновенное ускорение представляет собой наклон касательной. (а) Показано среднее ускорение

между временами

и

.Когда

, среднее ускорение приближается к мгновенному ускорению в момент времени t0. В виде (а) мгновенное ускорение показано для точки на кривой скорости при максимальной скорости. В этой точке мгновенное ускорение — это наклон касательной, равный нулю. В любое другое время наклон касательной — и, следовательно, мгновенное ускорение — не будет нулевым. (b) То же, что (a), но показано для мгновенного ускорения при минимальной скорости.

Чтобы проиллюстрировать эту концепцию, давайте рассмотрим два примера.Во-первых, простой пример показан с использованием (Рисунок) (b), графика зависимости скорости от времени (Рисунок), для графического определения ускорения. Этот график изображен на (Рисунок) (а), который представляет собой прямую линию. Соответствующий график ускорения в зависимости от времени находится по наклону скорости и показан на (Рисунок) (b). В этом примере функция скорости представляет собой прямую линию с постоянным наклоном, поэтому ускорение является постоянным. В следующем примере функция скорости имеет более сложную функциональную зависимость от времени.

Рис. 3.15. (a, b) График зависимости скорости от времени является линейным и имеет постоянный отрицательный наклон (a), который равен ускорению, показанному на (b).

Если мы знаем функциональную форму скорости, v ( t ), мы можем вычислить мгновенное ускорение a ( t ) в любой момент времени в движении, используя (рисунок).

Пример

Расчет мгновенного ускорения

Частица движется и ускоряется.Функциональная форма скорости

.

  1. Найдите функциональную форму ускорения.
  2. Найдите мгновенную скорость при t = 1, 2, 3 и 5 с.
  3. Найдите мгновенное ускорение при t = 1, 2, 3 и 5 с.
  4. Интерпретируйте результаты (c) с точки зрения направлений векторов ускорения и скорости.
Стратегия

Функциональную форму ускорения находим, взяв производную от функции скорости.Затем мы вычисляем значения мгновенной скорости и ускорения из заданных функций для каждой. Для части (d) нам нужно каждый раз сравнивать направления скорости и ускорения.

Решение
  1. ,

    ,

    ,

  2. ,

    ,

    ,

  3. При т = 1 с, скорость

    положительный, а ускорение положительное, поэтому скорость и ускорение имеют одно направление.Частица движется быстрее.

При т = 2 с, скорость увеличилась до

, где он максимальный, что соответствует моменту, когда ускорение равно нулю. Мы видим, что максимальная скорость достигается, когда наклон функции скорости равен нулю, что является просто нулем функции ускорения.

При т = 3 с, скорость

и ускорение отрицательное. Частица уменьшила свою скорость, и вектор ускорения отрицательный.Частица замедляется.

При т = 5 с, скорость

и ускорение становится все более отрицательным. Между моментами времени t = 3 с и t = 5 с частица уменьшила свою скорость до нуля, а затем стала отрицательной, таким образом изменив свое направление. Теперь частица снова ускоряется, но в противоположном направлении.

Мы можем видеть эти результаты графически на (Рисунок).

Рис. 3.16. (а) Скорость в зависимости от времени.Касательные линии указаны в моменты времени 1, 2 и 3 с. Наклоны касательных — это ускорения. При t = 3 с скорость положительная. При t = 5 с скорость отрицательна, что указывает на то, что частица поменяла направление на обратное. (б) Ускорение против времени. Сравнивая значения ускорений, представленные черными точками, с соответствующими наклонами касательных линий (наклон линий через черные точки) на (а), мы видим, что они идентичны.

Значение

Выполняя численный и графический анализ скорости и ускорения частицы, мы можем многое узнать о ее движении.Численный анализ дополняет графический анализ, давая полное представление о движении. Нуль функции ускорения соответствует максимуму скорости в этом примере. Также в этом примере, когда ускорение положительное и в том же направлении, что и скорость, скорость увеличивается. По мере того, как ускорение стремится к нулю, постепенно становясь отрицательным, скорость достигает максимума, после чего начинает уменьшаться. Если мы подождем достаточно долго, скорость также станет отрицательной, что указывает на изменение направления.Реальным примером такого движения является автомобиль, скорость которого увеличивается до максимальной, после чего он начинает замедляться, останавливается, а затем меняет направление движения.

Проверьте свое понимание

Самолет приземляется на взлетно-посадочной полосе, летящей на восток. Опишите его ускорение.

[показывать-ответ q = ”fs-id1168327963777 ″] Показать решение [/ показывать-ответ]

[скрытый-ответ a = ”fs-id1168327963777 ″]

Если принять за положительное значение восток, то самолет имеет отрицательное ускорение, потому что он ускоряется в сторону запада.Он также замедляется; его ускорение противоположно направлению его скорости.

[/ hidden-answer]

Ощущение ускорения

Вы, вероятно, привыкли испытывать ускорение, когда заходите в лифт или нажимаете на педаль газа в машине. Однако ускорение происходит со многими другими объектами в нашей Вселенной, с которыми мы не имеем прямого контакта. (Рисунок) представлено ускорение различных объектов. Мы можем видеть, что величины ускорений простираются на многие порядки.

Типичные значения ускорения (источник: Википедия: порядки величин (ускорение))
Разгон Значение (м / с 2 )
Скоростной поезд 0,25
Лифт 2
Гепард 5
Объект в свободном падении без сопротивления воздуха у поверхности Земли 9,8
Максимум космического челнока во время запуска 29
Пик парашютиста при нормальном раскрытии парашюта 59
Самолет F16 выходит из пикирования 79
Взрывное выброс сиденья с самолета 147
Ракета Sprint 982
Максимальное пиковое ускорение ракетных салазок 1540
Прыгающая блоха 3200
Бейсбольный удар битой 30 000
Захваты муравья-ловушки 1 000 000
Протон в большом адронном коллайдере

В этой таблице мы видим, что типичные ускорения сильно различаются для разных объектов и не имеют ничего общего с размером объекта или его массивностью.Ускорение также может сильно меняться со временем во время движения объекта. У дрэг-рейсинга большое ускорение сразу после старта, но затем оно уменьшается, когда транспортное средство достигает постоянной скорости. Его среднее ускорение может сильно отличаться от его мгновенного ускорения в определенный момент времени во время его движения. (Рисунок) графически сравнивает среднее ускорение с мгновенным ускорением для двух очень разных движений.

Рис. 3.17. Графики мгновенного ускорения в зависимости от времени для двух различных одномерных движений.(а) Ускорение меняется незначительно и всегда в одном и том же направлении, поскольку оно положительное. Среднее значение за интервал почти такое же, как и ускорение в любой момент времени. (b) Ускорение сильно различается, возможно, представляя пакет на конвейерной ленте почтового отделения, который ускоряется вперед и назад, когда он натыкается. В такой ситуации необходимо учитывать небольшие интервалы времени (например, от 0 до 1,0 с) с постоянным или почти постоянным ускорением.

Сводка

  • Ускорение — это скорость изменения скорости.Ускорение — это вектор; он имеет как величину, так и направление. Единица измерения ускорения в системе СИ — метр на секунду в квадрате.
  • Ускорение может быть вызвано изменением величины или направления скорости, либо и тем, и другим.
  • Мгновенное ускорение a ( t ) является непрерывной функцией времени и дает ускорение в любой конкретный момент во время движения. Он рассчитывается по производной функции скорости. Мгновенное ускорение — это наклон графика зависимости скорости от времени.
  • Отрицательное ускорение (иногда называемое замедлением) — это ускорение в отрицательном направлении в выбранной системе координат.

Концептуальные вопросы

Возможно ли, чтобы скорость была постоянной, когда ускорение не равно нулю?

[show-answer q = ”fs-id1168328025381 ″] Показать решение [/ show-answer]

[скрытый-ответ a = ”fs-id1168328025381 ″]

Нет, в одном измерении постоянная скорость требует нулевого ускорения.

[/ hidden-answer]

Может ли скорость быть постоянной, если ускорение не равно нулю? Объяснять.

Приведите пример, в котором скорость равна нулю, а ускорение — нет.

[show-answer q = ”fs-id11683282 ″] Показать решение [/ show-answer]

[скрытый-ответ a = ”fs-id11683282 ″]

Мяч подбрасывается в воздух, и его скорость равна нулю на вершине броска, но ускорение не равно нулю.

[/ hidden-answer]

Если поезд метро движется влево (имеет отрицательную скорость), а затем останавливается, в каком направлении он ускоряется? Ускорение положительное или отрицательное?

Знаки плюс и минус используются в одномерном движении для обозначения направления.Каков знак ускорения, уменьшающего величину отрицательной скорости? Положительной скорости?

[show-answer q = ”fs-id1168328228855 ″] Показать решение [/ show-answer]

[скрытый-ответ a = ”fs-id1168328228855 ″]

Плюс, минус

[/ hidden-answer]

Гепард может разогнаться от состояния покоя до скорости 30,0 м / с за 7,00 с. Какое у него ускорение?

[show-answer q = ”fs-id1168328195958 ″] Показать решение [/ show-answer]

[скрытый-ответ a = ”fs-id1168328195958 ″]

[/ hidden-answer]

Доктор.Джон Пол Стапп был офицером ВВС США, изучавшим влияние экстремального ускорения на человеческое тело. 10 декабря 1954 года Стапп проехал на ракетных санях, разогнавшись из состояния покоя до максимальной скорости 282 м / с (1015 км / ч) за 5,00 с и резко остановившись всего за 1,40 с. Вычислите его (а) ускорение в направлении его движения и (б) ускорение, противоположное его направлению движения. Выразите каждое значение кратным g (9,80 м / с 2 ), взяв его отношение к ускорению свободного падения.

Нарисуйте график зависимости ускорения от времени из следующего графика зависимости скорости от времени.

[show-answer q = ”1811 ″] Показать ответ [/ show-answer]
[hidden-answer a =” 1811 ″] [/ hidden-answer]

Пассажир выезжает на машине из гаража с ускорением 1,40 м / с. 2 . а) Сколько времени ей нужно, чтобы набрать скорость 2,00 м / с? (b) Если она затем тормозит до остановки за 0,800 с, каково ее ускорение?

Предположим, межконтинентальная баллистическая ракета переходит из состояния покоя в суборбитальную скорость 6.50 км / с за 60,0 с (фактическая скорость и время засекречены). Каково его среднее ускорение в метрах в секунду и кратное g (9,80 м / с 2 )?

[показывать-ответ q = ”fs-id1168325667515 ″] Показать решение [/ показывать-ответ]

[скрытый-ответ a = ”fs-id1168325667515 ″]

[/ hidden-answer]

Самолет, взлетая с места, движется по взлетно-посадочной полосе с постоянным ускорением в течение 18 с, а затем взлетает со скоростью 60 м / с.Какое среднее ускорение самолета?

Глоссарий

среднее ускорение
скорость изменения скорости; изменение скорости с течением времени
мгновенное ускорение
ускорение в определенный момент времени

2.3 Графики положения в зависимости от времени — физика

Построение графика в зависимости от времени

График, как и картинка, стоит тысячи слов.Графики содержат не только числовую информацию, они также показывают взаимосвязь между физическими величинами. В этом разделе мы исследуем кинематику, анализируя графики положения во времени.

Графики в этом тексте имеют перпендикулярные оси, одна горизонтальная, а другая вертикальная. Когда две физические величины наносятся друг на друга, горизонтальная ось обычно считается независимой переменной, а вертикальная ось — зависимой переменной. В алгебре вы бы назвали горизонтальную ось осью x , а вертикальную ось — осью y .Как показано на рис. 2.10, прямолинейный граф имеет общий вид y = mx + by = mx + b.

Здесь м. — уклон, определяемый как подъем, деленный на пробег (как показано на рисунке) прямой линии. Буква b представляет собой точку пересечения линии y , которая является точкой, в которой линия пересекает вертикальную ось y . С точки зрения физической ситуации в реальном мире эти величины будут иметь особое значение, как мы увидим ниже. (Рисунок 2.10.)

Рисунок 2.10 На диаграмме изображен прямолинейный график. Уравнение прямой: y равно mx + b .

В физике время обычно является независимой переменной. Говорят, что от него зависят другие величины, такие как смещение. График положения в зависимости от времени, следовательно, будет иметь положение на вертикальной оси (зависимая переменная) и время на горизонтальной оси (независимая переменная). В этом случае, к чему будут относиться наклон и пересечение y ? Давайте вернемся к нашему первоначальному примеру при изучении расстояния и смещения.

Дорога в школу находилась в 5 км от дома. Предположим, поездка заняла 10 минут, и ваш родитель все это время вел машину с постоянной скоростью. График зависимости положения от времени для этого участка пути будет выглядеть так, как показано на рисунке 2.11.

Рис. 2.11. Показан график зависимости положения от времени на дорогу в школу. Как бы выглядел график, если бы мы добавили обратный путь?

Как мы уже говорили, d 0 = 0, потому что мы называем домой наш O и начинаем вычисление оттуда.На рисунке 2.11 линия также начинается с d = 0. Это b в нашем уравнении для прямой. Нашей исходной позицией на графике зависимости положения от времени всегда является место, где график пересекает ось x при t = 0. Каков наклон? Подъем — это изменение положения (то есть смещение), а ход — это изменение во времени. Это отношение также можно записать

Это соотношение было тем, как мы определили среднюю скорость.Следовательно, наклон на графике d против t — это средняя скорость.

Советы для успеха

Иногда, как в случае, когда мы строим график как поездки в школу, так и обратного пути, поведение графика выглядит по-разному в разные промежутки времени. Если график выглядит как серия прямых линий, то вы можете рассчитать среднюю скорость для каждого временного интервала, посмотрев на наклон. Если вы затем захотите рассчитать среднюю скорость для всей поездки, вы можете сделать средневзвешенное значение.

Давайте посмотрим на другой пример. На рис. 2.12 показан график положения в зависимости от времени для реактивного автомобиля на очень плоском высохшем дне озера в Неваде.

Рис. 2.12 На диаграмме показан график положения в зависимости от времени для автомобиля с реактивным двигателем на солончаках Бонневиль.

Используя соотношение между зависимыми и независимыми переменными, мы видим, что наклон на графике на рисунке 2.12 — это средняя скорость, v avg , а точка пересечения — смещение в нулевой момент времени, то есть d 0 .Подставляя эти символы в y = mx + b , получаем

или

Таким образом, график положения в зависимости от времени дает общую взаимосвязь между перемещением, скоростью и временем, а также дает подробную числовую информацию о конкретной ситуации. Из рисунка видно, что автомобиль занимает позицию 400 м при т = 0 с, 650 м при т = 1,0 с и так далее. И мы также можем узнать о скорости объекта.

Поддержка учителей

Поддержка учителей
Демонстрация учителей

Помогите учащимся узнать, какие графики смещения отличаются от графиков.время похоже.

[Визуальный] Установите дозатор.

  1. Если вы можете найти машину с дистанционным управлением, попросите одного ученика записать время, когда вы отправляете машину вперед вдоль ручки, затем назад, затем снова вперед с постоянной скоростью.
  2. Возьмите записанное время и изменение положения и сложите их вместе.
  3. Попросите студентов научить вас рисовать график зависимости положения от времени.

Каждый отрезок пути должен представлять собой прямую линию с разным уклоном.Участки, по которым машина двигалась вперед, должны иметь положительный наклон. Та часть, где он движется назад, будет иметь отрицательный наклон.

[OL] Спросите, влияет ли на график место, которое они принимают за ноль .

[AL] Реально ли нарисовать любой график положения, который начинается в состоянии покоя, без какой-либо кривой? Почему в некоторых сценариях можно пренебречь кривой?

[Все] Обсудите, что можно обнаружить на этом графике. Учащиеся должны уметь считывать чистое смещение, но они также могут использовать график для определения общего пройденного расстояния.Затем спросите, как скорость или скорость отражаются на этом графике. Посоветуйте учащимся увидеть, что крутизна линии (уклона) является мерой скорости, а направление уклона — направлением движения.

[AL] Некоторые студенты могут понять, что кривая на линии представляет собой своего рода наклон наклона, предварительный просмотр ускорения, о котором они узнают в следующей главе.

Snap Lab

Построение графика движения

В этом упражнении вы отпустите мяч по наклонной плоскости и построите график зависимости смещения мяча от смещения.время.

  • Выберите открытое место с большим пространством, чтобы было меньше шансов споткнуться или упасть из-за катящихся шаров.
  • 1 мяч
  • 1 доска
  • 2 или 3 книги
  • 1 секундомер
  • 1 рулетка
  • 6 штук малярной ленты
  • 1 миллиметровая бумага
  • 1 карандаш

Процедура

  1. Постройте пандус, поместив один конец доски поверх стопки книг.При необходимости отрегулируйте местоположение так, чтобы не было препятствий на прямом пути от нижней части пандуса до следующих 3 м.
  2. Отметьте расстояния 0,5 м, 1,0 м, 1,5 м, 2,0 м, 2,5 м и 3,0 м от нижней части пандуса. Напишите расстояния на ленте.
  3. Пусть один человек возьмет на себя роль экспериментатора. Этот человек выпустит мяч с вершины рампы. Если мяч не достигает отметки 3,0 м, увеличьте наклон пандуса, добавив еще одну книгу.При необходимости повторите этот шаг.
  4. Попросите экспериментатора выпустить мяч. Попросите второго человека, таймера, начать отсчет времени испытания, когда мяч достигнет нижней части рампы, и остановить отсчет, когда мяч достигнет 0,5 м. Попросите третьего человека, записывающего устройства, записать время в таблицу данных.
  5. Повторите шаг 4, останавливая раз на расстоянии 1,0 м, 1,5 м, 2,0 м, 2,5 м и 3,0 м от нижней части пандуса.
  6. Используйте свои измерения времени и смещения, чтобы составить позицию vs.временной график движения мяча.
  7. Повторите шаги с 4 по 6 с разными людьми, которые берут на себя роли экспериментатора, таймера и записывающего устройства. Получаете ли вы одинаковые значения измерений независимо от того, кто выпускает мяч, измеряет время или записывает результат? Обсудите возможные причины расхождений, если таковые имеются.

Проверка захвата

Верно или неверно: средняя скорость мяча будет меньше средней скорости мяча.

  1. Истинно
  2. Ложь

Поддержка учителя

Поддержка учителя

[BL] [OL] Подчеркните, что движение в этой лабораторной работе — это движение мяча, катящегося по полу.Спросите студентов, где должен быть ноль.

[AL] Спросите студентов, как бы выглядел график, если бы они начали отсчет времени вверху по сравнению с основанием пандуса. Почему график должен выглядеть иначе? Чем может объясняться разница?

[BL] [OL] Попросите учащихся сравнить графики, построенные с разными людьми, выполняющими разные роли. Попросите их определить и сравнить среднюю скорость для каждого интервала. Каковы были абсолютные различия в скоростях и каковы были различия в процентах? Оказываются ли различия случайными или существуют систематические различия? Почему могут существовать систематические различия между двумя наборами измерений с разными людьми в каждой роли?

[BL] [OL] Попросите учащихся сравнить графики, построенные с разными людьми, выполняющими разные роли.Попросите их определить и сравнить среднюю скорость для каждого интервала. Каковы были абсолютные различия в скоростях и каковы были различия в процентах? Оказываются ли различия случайными или существуют систематические различия? Почему могут существовать систематические различия между двумя наборами измерений с разными людьми в каждой роли?

Решение задач с использованием графиков положения и времени

Итак, как мы можем использовать графики для решения таких задач, как скорость?

Рабочий пример

Использование графика положения и времени для расчета средней скорости: Jet Car

Найдите среднюю скорость автомобиля, положение которого показано на Рисунке 1.13.

Стратегия

Наклон графика d против t — это средняя скорость, поскольку наклон равен подъему за пробег.

наклон = ΔdΔt = vsсклон = ΔdΔt = v

2,7

Поскольку наклон здесь постоянный, любые две точки на графике могут использоваться для определения наклона.

Решение

  1. Выберите две точки на линии. В этом случае мы выбираем точки, помеченные на графике: (6,4 с, 2000 м) и (0,50 с, 525 м). (Обратите внимание, однако, что вы можете выбрать любые две точки.)
  2. Подставьте значения d и t выбранных точек в уравнение. Помните, что при вычислении изменения (Δ) мы всегда используем конечное значение минус начальное значение.
    v = ΔdΔt = 2000 м − 525 м6,4 с − 0,50 с = 250 м / с, v = ΔdΔt = 2000 м − 525 м6,4 с − 0,50 с = 250 м / с,

    2,8

Обсуждение

Это впечатляюще высокая сухопутная скорость (900 км / ч или около 560 миль / ч): намного больше, чем типичное ограничение скорости на шоссе, равное 27 м / с или 96 км / ч, но значительно ниже рекордных 343 м. / с или 1234 км / ч, установленный в 1997 году.

Teacher Support

Teacher Support

Если график положения представляет собой прямую линию, то единственное, что ученики должны знать для расчета средней скорости, — это наклон линии, подъем / бег. Они могут использовать любые наиболее удобные точки на линии.

А что, если график позиции сложнее прямой? Что, если объект ускоряется или поворачивается и движется назад? Можем ли мы выяснить что-нибудь о его скорости из графика такого движения? Давайте еще раз посмотрим на реактивный автомобиль.График на рис. 2.13 показывает его движение по мере набора скорости после запуска из состояния покоя. Время для этого движения начинается с нуля (как если бы оно измерялось секундомером), а смещение и скорость изначально составляют 200 м и 15 м / с соответственно.

Рис. 2.13 На диаграмме показан график положения автомобиля с реактивным двигателем в течение периода времени, когда он набирает скорость. Наклон графика зависимости расстояния от времени — это скорость. Это показано в двух точках. Мгновенная скорость в любой точке — это наклон касательной в этой точке.

Рис. 2.14 Реактивный автомобиль ВВС США едет по рельсовому пути. (Мэтт Тростле, Flickr)

График положения в зависимости от времени на рис. 2.13 представляет собой кривую, а не прямую линию. Наклон кривой становится более крутым с течением времени, показывая, что скорость увеличивается с течением времени. Наклон в любой точке графика зависимости положения от времени — это мгновенная скорость в этой точке. Его можно найти, проведя прямую касательную к кривой в интересующей точке и взяв наклон этой прямой.Касательные линии показаны для двух точек на рисунке 2.13. Средняя скорость — это чистое смещение, деленное на пройденное время.

Рабочий пример

Использование графика положения и времени для расчета средней скорости: реактивный автомобиль, дубль

Рассчитайте мгновенную скорость реактивного автомобиля за время 25 с, определив наклон касательной в точке Q на рисунке 2.13.

Стратегия

Наклон кривой в точке равен наклону прямой, касательной к кривой в этой точке.

Решение

  1. Найдите касательную к кривой при t = 25 st = 25 с.
  2. Определите конечные точки касательной. Они соответствуют положению 1300 м за 19 с и положению 3120 м за 32 с.
  3. Подставьте эти конечные точки в уравнение, чтобы найти наклон, v .
    уклон = vQ = ΔdQΔtQ = (3120−1300) м (32−19) s = 1820 м13 s = 140 м / с уклон = vQ = ΔdQΔtQ = (3120−1300) м (32−19) s = 1820 м13 s = 140 м / с

    2.9

Обсуждение

Таким образом можно получить весь график v и t .

Поддержка учителя

Поддержка учителя

Изогнутая линия — более сложный пример. Определите касательную как линию, которая касается кривой только в одной точке. Покажите, что, когда прямая линия меняет свой угол рядом с кривой, она на самом деле несколько раз ударяет по кривой в основании, но только одна линия никогда не соприкасается. Эта линия образует прямой угол с радиусом кривизны, но на этом уровне они могут просто смотреть на нее.Наклон этой линии дает мгновенную скорость. Самая полезная часть этой строки состоит в том, что учащиеся могут определить, когда скорость увеличивается, уменьшается, положительная, отрицательная и нулевая.

[AL] Вы можете найти мгновенную скорость в каждой точке графика, и если вы изобразите каждую из этих точек, вы получите график скорости.

3.4 Движение с постоянным ускорением — University Physics Volume 1

Учебные цели

К концу этого раздела вы сможете:

  • Определите, какие уравнения движения следует использовать для решения неизвестных.
  • Используйте соответствующие уравнения движения для решения задачи преследования двух тел.

Можно предположить, что чем больше ускорение, скажем, у автомобиля, удаляющегося от знака «Стоп», тем больше смещение автомобиля за данный момент времени. Но мы не разработали конкретное уравнение, которое связывает ускорение и смещение. В этом разделе мы рассмотрим некоторые удобные уравнения кинематических отношений, начиная с определений смещения, скорости и ускорения.Сначала мы исследуем движение одного объекта, называемого движением одного тела. Затем мы исследуем движение двух объектов, называемое задачами преследования двух тел.

Обозначение

Сначала сделаем несколько упрощений в обозначениях. Принятие начального времени равным нулю, как если бы время измерялось секундомером, является большим упрощением. Поскольку прошедшее время равно Δt = tf − t0Δt = tf − t0, принятие t0 = 0t0 = 0 означает, что Δt = tfΔt = tf, последнее время на секундомере. Когда начальное время принимается равным нулю, мы используем индекс 0 для обозначения начальных значений положения и скорости.То есть x0x0 — это начальная позиция , а v0v0 — начальная скорость . Мы не ставим индексы на окончательные значения. То есть t — это последнее время , x — это конечная позиция , а v — это конечная скорость . Это дает более простое выражение для затраченного времени: Δt = tΔt = t. Это также упрощает выражение для смещения x , которое теперь равно Δx = x − x0Δx = x − x0. Кроме того, это упрощает выражение для изменения скорости, которое теперь равно Δv = v − v0Δv = v − v0.Подводя итог, используя упрощенные обозначения, с начальным временем, принятым равным нулю,

Δt = tΔx = x − x0Δv = v − v0, Δt = tΔx = x − x0Δv = v − v0,

, где нижний индекс 0 обозначает начальное значение, а отсутствие нижнего индекса означает конечное значение в любом рассматриваемом движении.

Теперь мы делаем важное предположение, что ускорение является постоянным . Это предположение позволяет нам избегать использования расчетов для определения мгновенного ускорения. Поскольку ускорение постоянно, среднее и мгновенное ускорения равны, то есть

a– = a = постоянная.a– = a = постоянная.

Таким образом, мы можем использовать символ a для ускорения в любое время. Предположение, что ускорение является постоянным, не серьезно ограничивает ситуации, которые мы можем изучить, и не ухудшает точность нашего лечения. Во-первых, ускорение всегда равно в большом количестве ситуаций. Кроме того, во многих других ситуациях мы можем точно описать движение, приняв постоянное ускорение, равное среднему ускорению для этого движения. Наконец, для движения, во время которого ускорение резко меняется, например, когда автомобиль разгоняется до максимальной скорости, а затем тормозит до остановки, движение можно рассматривать в отдельных частях, каждая из которых имеет собственное постоянное ускорение.

Смещение и положение от скорости

Чтобы получить наши первые два уравнения, мы начнем с определения средней скорости:

Замена упрощенных обозначений для ΔxΔx и ΔtΔt дает

v– = x − x0t.v– = x − x0t.

Решение для x дает нам

x = x0 + v – t, x = x0 + v – t,

3,10

, где средняя скорость

v– = v0 + v2.v– = v0 + v2.

3,11

Уравнение v– = v0 + v2v– = v0 + v2 отражает тот факт, что при постоянном ускорении v – v– представляет собой простое среднее значение начальной и конечной скоростей.Рисунок 3.18 графически иллюстрирует эту концепцию. В части (а) рисунка ускорение является постоянным, а скорость увеличивается с постоянной скоростью. Средняя скорость в течение 1-часового интервала от 40 км / ч до 80 км / ч составляет 60 км / ч:

v– = v0 + v2 = 40 км / ч + 80 км / ч3 = 60 км / ч v– = v0 + v2 = 40 км / ч + 80 км / ч3 = 60 км / ч.

В части (b) ускорение не является постоянным. В течение 1-часового интервала скорость ближе к 80 км / ч, чем к 40 км / ч. Таким образом, средняя скорость больше, чем в части (а).

Рисунок 3.18 (a) График зависимости скорости от времени с постоянным ускорением, показывающий начальную и конечную скорости v0andvv0andv.Средняя скорость равна 12 (v0 + v) = 60 км / ч22 (v0 + v) = 60 км / ч. (б) График зависимости скорости от времени с изменением ускорения со временем. Средняя скорость не равна 12 (v0 + v) 12 (v0 + v), но превышает 60 км / ч.

Решение для окончательной скорости по ускорению и времени

Мы можем вывести еще одно полезное уравнение, манипулируя определением ускорения:

Подстановка упрощенных обозначений для ΔvΔv и ΔtΔt дает

а = v − v0t (константа). a = v − v0t (константа).

Решение для v дает

v = v0 + at (constanta). v = v0 + at (constanta).

3,12

Пример 3,7

Расчет конечной скорости

Самолет приземляется с начальной скоростью 70,0 м / с, а затем набирает скорость, противоположную движению, со скоростью 1,50 м / с 2 за 40,0 с. Какова его конечная скорость?

Стратегия

Сначала мы идентифицируем известные: v0 = 70 м / с, a = -1,50 м / с2, t = 40sv0 = 70 м / с, a = -1,50 м / с2, t = 40 с.

Во-вторых, мы идентифицируем неизвестное; в данном случае это конечная скорость vfvf.

Наконец, мы определяем, какое уравнение использовать. Для этого мы выясняем, какое кинематическое уравнение дает неизвестное в терминах известных. Мы вычисляем конечную скорость, используя уравнение 3.12, v = v0 + atv = v0 + at.

Решение

Подставьте известные значения и решите:
v = v0 + при = 70,0 м / с + (- 1,50 м / с2) (40,0 с) = 10,0 м / с v = v0 + при = 70,0 м / с + (- 1,50 м / с2) (40,0 с) = 10,0 м / с.

Рисунок 3.19 представляет собой эскиз, на котором показаны векторы ускорения и скорости.

Рис. 3.19 Самолет приземляется с начальной скоростью 70.0 м / с и замедляется до конечной скорости 10,0 м / с, прежде чем направиться к терминалу. Обратите внимание, что ускорение отрицательное, потому что его направление противоположно его скорости, которая положительна.

Значение

Конечная скорость намного меньше начальной скорости, требуемой при замедлении, но все же положительная (см. Рисунок). С реактивными двигателями обратная тяга может поддерживаться достаточно долго, чтобы остановить самолет и начать движение назад, на что указывает отрицательная конечная скорость, но в данном случае это не так.

Помимо полезности при решении задач, уравнение v = v0 + atv = v0 + at дает нам представление о взаимосвязях между скоростью, ускорением и временем. Мы видим, например, что

  • Конечная скорость зависит от того, насколько велико ускорение и как долго оно длится
  • Если ускорение равно нулю, то конечная скорость равна начальной скорости ( v = v 0 ), как и ожидалось (другими словами, скорость постоянна)
  • Если a отрицательно, то конечная скорость меньше начальной скорости

Все эти наблюдения соответствуют нашей интуиции.Обратите внимание, что всегда полезно исследовать основные уравнения в свете нашей интуиции и опыта, чтобы убедиться, что они действительно точно описывают природу.

Решение для конечного положения с постоянным ускорением

Мы можем объединить предыдущие уравнения, чтобы найти третье уравнение, которое позволяет нам вычислить окончательное положение объекта, испытывающего постоянное ускорение. Начнем с

Добавление v0v0 к каждой стороне этого уравнения и деление на 2 дает

v0 + v2 = v0 + 12at.v0 + v2 = v0 + 12at.

Так как v0 + v2 = v – v0 + v2 = v– для постоянного ускорения, имеем

v– = v0 + 12at.v– = v0 + 12at.

Теперь мы подставляем это выражение для v – v– в уравнение для смещения, x = x0 + v – tx = x0 + v – t, что дает

x = x0 + v0t + 12at2 (константа). x = x0 + v0t + 12at2 (константа).

3,13

Пример 3.8

Расчет смещения ускоряющегося объекта

Драгстеры могут развивать среднее ускорение 26,0 м / с 2 . Предположим, драгстер ускоряется из состояния покоя на 5.56 с Рисунок 3.20. Как далеко он пролетит за это время?

Рисунок 3.20. Пилот Top Fuel в армии США Тони «Сержант» Шумахер начинает гонку с контролируемого выгорания. (Источник: подполковник Уильям Термонд. Фотография предоставлена ​​армией США.)

Стратегия

Сначала нарисуем набросок Рис. 3.21. Нас просят найти смещение, которое составляет x , если мы примем x0x0 равным нулю. (Думайте о x0x0 как о стартовой линии гонки. Она может быть где угодно, но мы называем ее нулевой и измеряем все остальные позиции относительно нее.) Мы можем использовать уравнение x = x0 + v0t + 12at2x = x0 + v0t + 12at2, когда мы идентифицируем v0v0, aa и t из постановки задачи.

Рис. 3.21. Эскиз разгоняющегося драгстера.

Решение

Во-первых, нам нужно определить известные. Запуск из состояния покоя означает, что v0 = 0v0 = 0, a задается как 26,0 м / с 2 и t задается как 5,56 с.

Во-вторых, мы подставляем известные значения в уравнение, чтобы найти неизвестное:

х = х0 + v0t + 12at2.х = х0 + v0t + 12at2.

Поскольку начальное положение и скорость равны нулю, это уравнение упрощается до

Подстановка идентифицированных значений на и t дает

x = 12 (26,0 м / с2) (5,56 с) 2 = 402 м. x = 12 (26,0 м / с2) (5,56 с) 2 = 402 м.

Значение

Если мы переведем 402 м в мили, мы обнаружим, что пройденное расстояние очень близко к четверти мили, стандартному расстоянию для дрэг-рейсинга. Итак, наш ответ разумный. Это впечатляющий рабочий объем, который можно покрыть всего за 5.56 с, но первоклассные драгстеры могут преодолеть четверть мили даже за меньшее время, чем это. Если бы драгстеру была присвоена начальная скорость, это добавило бы еще один член в уравнение расстояния. Если в уравнении использовать те же ускорение и время, пройденное расстояние будет намного больше.

Что еще мы можем узнать, исследуя уравнение x = x0 + v0t + 12at2? X = x0 + v0t + 12at2? Мы видим следующие отношения:

  • Смещение зависит от квадрата прошедшего времени, когда ускорение не равно нулю.В примере 3.8 драгстер преодолевает только четверть общего расстояния за первую половину прошедшего времени.
  • Если ускорение равно нулю, то начальная скорость равна средней скорости (v0 = v -) (v0 = v–), и x = x0 + v0t + 12at2becomesx = x0 + v0t.x = x0 + v0t + 12at2becomesx = x0 + v0t.

Решение окончательной скорости по расстоянию и ускорению

Четвертое полезное уравнение может быть получено путем другой алгебраической обработки предыдущих уравнений. Если мы решим v = v0 + atv = v0 + at для t , мы получим

Подставляя это и v– = v0 + v2v– = v0 + v2 в x = x0 + v – tx = x0 + v – t, получаем

v2 = v02 + 2a (x − x0) (константа).v2 = v02 + 2a (x − x0) (константа).

3,14

Пример 3.9

Расчет конечной скорости

Рассчитайте конечную скорость драгстера в Примере 3.8 без использования информации о времени.

Стратегия

Уравнение v2 = v02 + 2a (x − x0) v2 = v02 + 2a (x − x0) идеально подходит для этой задачи, поскольку оно связывает скорости, ускорение и смещение и не требует информации о времени.

Решение

Сначала мы идентифицируем известные значения. Мы знаем, что v 0 = 0, поскольку драгстер запускается из состояния покоя.Мы также знаем, что x x 0 = 402 м (это был ответ в примере 3.8). Среднее ускорение было равно , = 26,0 м / с 2 .

Во-вторых, мы подставляем известные в уравнение v2 = v02 + 2a (x − x0) v2 = v02 + 2a (x − x0) и решаем относительно v :

v2 = 0 + 2 (26,0 м / с2) (402 м). v2 = 0 + 2 (26,0 м / с2) (402 м).

Таким образом,

v2 = 2,09 × 104 м2 / с2 v = 2,09 × 104 м2 / с2 = 145 м / с. v2 = 2,09 × 104 м2 / с2v = 2,09 × 104 м2 / с2 = 145 м / с.

Значение

Скорость 145 м / с составляет около 522 км / ч или около 324 миль / ч, но даже эта головокружительная скорость не достигает рекорда для четверти мили.Также обратите внимание, что квадратный корень имеет два значения; мы взяли положительное значение, чтобы указать скорость в том же направлении, что и ускорение.

Изучение уравнения v2 = v02 + 2a (x − x0) v2 = v02 + 2a (x − x0) может дать дополнительное понимание общих соотношений между физическими величинами:

  • Конечная скорость зависит от величины ускорения и расстояния, на котором оно действует.
  • При фиксированном ускорении автомобиль, который едет вдвое быстрее, просто не останавливается на удвоенном расстоянии.Чтобы остановиться, нужно гораздо дальше. (Вот почему у нас есть зоны с пониженной скоростью возле школ.)

Объединение уравнений

В следующих примерах мы продолжаем исследовать одномерное движение, но в ситуациях, требующих немного большего количества алгебраических манипуляций. Примеры также дают представление о методах решения проблем. Следующее примечание предназначено для облегчения поиска необходимых уравнений. Имейте в виду, что эти уравнения не являются независимыми. Во многих ситуациях у нас есть два неизвестных, и нам нужно два уравнения из набора для решения неизвестных.Для решения данной ситуации нам нужно столько уравнений, сколько неизвестных.

Сводка кинематических уравнений (константа

a )

х = х0 + v0t + 12at2x = x0 + v0t + 12at2
v2 = v02 + 2a (x − x0) v2 = v02 + 2a (x − x0)

Прежде чем перейти к примерам, давайте более внимательно рассмотрим некоторые уравнения, чтобы увидеть поведение ускорения при экстремальных значениях. Переставляя уравнение 3.12, получаем

Из этого мы видим, что в течение конечного времени, если разница между начальной и конечной скоростями мала, ускорение невелико, приближаясь к нулю в пределе, когда начальная и конечная скорости равны.Напротив, в пределе t → 0t → 0 при конечной разности начальной и конечной скоростей ускорение становится бесконечным.

Аналогичным образом, переставляя уравнение 3.14, мы можем выразить ускорение в терминах скоростей и смещения:

а = v2-v022 (х-х0). а = v2-v022 (х-х0).

Таким образом, при конечной разнице между начальной и конечной скоростями ускорение становится бесконечным, в пределе смещение приближается к нулю. Ускорение приближается к нулю в пределе, разница в начальной и конечной скоростях приближается к нулю для конечного смещения.

Пример 3.10

Как далеко уезжает машина?

На сухом бетоне автомобиль может ускоряться против движения со скоростью 7,00 м / с 2 , тогда как на мокром бетоне он может ускоряться против движения со скоростью всего 5,00 м / с 2 . Найдите расстояния, необходимые для остановки автомобиля, движущегося со скоростью 30,0 м / с (около 110 км / ч) по (а) сухому бетону и (б) мокрому бетону. (c) Повторите оба вычисления и найдите смещение от точки, где водитель видит, что светофор становится красным, принимая во внимание время его реакции, равное 0.500 с, чтобы нажать на педаль тормоза.

Стратегия

Для начала нам нужно нарисовать набросок Рис. 3.22. Чтобы определить, какие уравнения лучше всего использовать, нам нужно перечислить все известные значения и точно определить, что нам нужно решить.

Рис. 3.22. Пример эскиза для визуализации ускорения, противоположного движению и тормозному пути автомобиля.

Решение
  1. Во-первых, нам нужно определить известные и то, что мы хотим решить. Мы знаем, что против 0 = 30.0 м / с, v = 0 и a = -7,00 м / с 2 ( a отрицательно, потому что оно находится в направлении, противоположном скорости). Возьмем x 0 равным нулю. Ищем смещение ΔxΔx, или x x 0 .
    Во-вторых, мы определяем уравнение, которое поможет нам решить проблему. Лучшее уравнение для использования —
    v2 = v02 + 2a (x − x0). v2 = v02 + 2a (x − x0).
    Это уравнение лучше всего, потому что оно включает только одно неизвестное, x .Нам известны значения всех других переменных в этом уравнении. (Другие уравнения позволили бы нам решить для x , но они требуют, чтобы мы знали время остановки, t , которое мы не знаем. Мы могли бы использовать их, но это потребовало бы дополнительных вычислений.)
    В-третьих, мы изменим уравнение, чтобы найти x :
    x − x0 = v2 − v022ax − x0 = v2 − v022a
    и подставляем известные значения:
    x − 0 = 02− (30,0 м / с) 22 (−7,00 м / с2). x − 0 = 02− (30,0 м / с) 22 (−7,00 м / с2).
    Таким образом,
    x = 64,3 м по сухому бетону. x = 64.3м по сухому бетону.
  2. Эта часть может быть решена точно так же, как (а). Единственное отличие состоит в том, что ускорение составляет −5,00 м / с 2 . Результат
    xwet = 90,0 м по мокрому бетону. xwet = 90,0 м по мокрому бетону.
  3. Когда водитель реагирует, тормозной путь такой же, как в (a) и (b) для сухого и влажного бетона. Итак, чтобы ответить на этот вопрос, нам нужно вычислить, как далеко проехал автомобиль за время реакции, а затем добавить это время ко времени остановки. Разумно предположить, что скорость остается постоянной в течение времени реакции водителя.
    Для этого мы снова определяем известные и то, что мы хотим решить. Мы знаем, что v– = 30,0 м / sv– = 30,0 м / с, treaction = 0,500streaction = 0,500s и areaction = 0areaction = 0. Примем x0-реакцию x0-реакцию равной нулю. Ищем xreactionxreaction.
    Во-вторых, как и раньше, мы определяем лучшее уравнение для использования. В этом случае x = x0 + v – tx = x0 + v – t работает хорошо, потому что единственное неизвестное значение — x , что мы и хотим найти.
    В-третьих, мы подставляем известные для решения уравнения:
    х = 0 + (30.0 м / с) (0,500 с) = 15,0 м. X = 0 + (30,0 м / с) (0,500 с) = 15,0 м.
    Это означает, что автомобиль проезжает 15,0 м, в то время как водитель реагирует, в результате чего общее смещение в двух случаях с сухим и мокрым бетоном на 15,0 м больше, чем если бы он реагировал мгновенно.
    Наконец, мы добавляем смещение во время реакции к смещению при торможении (рис. 3.23),
    xbraking + xreaction = xtotal, xbraking + xreaction = xtotal,
    и найдите (а) равным 64,3 м + 15,0 м = 79,3 м в сухом состоянии и (б) равным 90,0 м + 15,0 м = 105 м во влажном состоянии.

Рисунок 3.23 Расстояние, необходимое для остановки автомобиля, сильно различается в зависимости от дорожных условий и времени реакции водителя. Здесь показаны значения тормозного пути для сухого и мокрого покрытия, рассчитанные в этом примере для автомобиля, движущегося со скоростью 30,0 м / с. Также показаны общие расстояния, пройденные от точки, когда водитель впервые видит, что свет загорается красным, при условии, что время реакции составляет 0,500 с.

Значение

Смещения, обнаруженные в этом примере, кажутся разумными для остановки быстро движущегося автомобиля.Остановка автомобиля на мокром асфальте должна длиться дольше, чем на сухом. Интересно, что время реакции значительно увеличивает смещения, но более важен общий подход к решению проблем. Мы идентифицируем известные и определяемые величины, а затем находим соответствующее уравнение. Если существует более одного неизвестного, нам нужно столько независимых уравнений, сколько неизвестных необходимо решить. Часто есть несколько способов решить проблему. Фактически, различные части этого примера могут быть решены другими методами, но представленные здесь решения являются самыми короткими.

Пример 3.11

Время расчета

Предположим, автомобиль въезжает в движение по автостраде на съезде длиной 200 м. Если его начальная скорость составляет 10,0 м / с, а ускорение составляет 2,00 м / с 2 , сколько времени потребуется машине, чтобы преодолеть 200 м по рампе? (Такая информация может быть полезна транспортному инженеру.)

Стратегия

Сначала мы рисуем набросок Рис. 3.24. Нам предлагается решить за время т . Как и раньше, мы идентифицируем известные величины, чтобы выбрать удобную физическую связь (то есть уравнение с одной неизвестной, t .)

Рис. 3.24. Эскиз автомобиля, ускоряющегося на съезде с автострады.

Решение

Опять же, мы определяем известные и то, что мы хотим решить. Мы знаем, что x0 = 0, x0 = 0,
v0 = 10 м / с, a = 2,00 м / с2v0 = 10 м / с, a = 2,00 м / с2 и x = 200 м.

Нужно решить для т . Уравнение x = x0 + v0t + 12at2x = x0 + v0t + 12at2 работает лучше всего, потому что единственной неизвестной в уравнении является переменная t , которую нам нужно решить. Из этого понимания мы видим, что когда мы вводим известные значения в уравнение, мы получаем квадратное уравнение.

Нам нужно изменить уравнение, чтобы найти t , затем подставив известные значения в уравнение:

200 м = 0 м + (10,0 м / с) t + 12 (2,00 м / с2) t2. 200 м = 0 м + (10,0 м / с) t + 12 (2,00 м / с2) t2.

Затем мы упрощаем уравнение. Единицы измерения отменяются, потому что они есть в каждом члене. Мы можем получить единицы секунд для отмены, взяв t = t s, где t — величина времени, а s — единица измерения. Остается

Затем мы используем формулу корней квадратного уравнения, чтобы найти t ,

t2 + 10t − 200 = 0t = −b ± b2−4ac2a, t2 + 10t − 200 = 0t = −b ± b2−4ac2a,

, что дает два решения: t = 10.0 и t = -20,0. Отрицательное значение времени неразумно, так как это будет означать, что событие произошло за 20 секунд до начала движения. Мы можем отказаться от этого решения. Таким образом,

Значение

Всякий раз, когда уравнение содержит неизвестный квадрат, есть два решения. В некоторых проблемах имеют смысл оба решения; в других случаях разумно только одно решение. Ответ 10,0 с кажется разумным для типичной автострады на съезде.

Проверьте свое понимание 3.5

Ракета ускоряется со скоростью 20 м / с. 2 во время пуска.Сколько времени нужно, чтобы ракета достигла скорости 400 м / с?

Пример 3.12

Ускорение космического корабля

Космический корабль покинул орбиту Земли и направляется к Луне. Разгоняется со скоростью 20 м / с 2 за 2 мин и преодолевает расстояние в 1000 км. Каковы начальная и конечная скорости космического корабля?

Стратегия

Нас просят найти начальную и конечную скорости космического корабля. Глядя на кинематические уравнения, мы видим, что одно уравнение не дает ответа.Мы должны использовать одно кинематическое уравнение для решения одной из скоростей и подставить его в другое кинематическое уравнение, чтобы получить вторую скорость. Таким образом, мы решаем два кинематических уравнения одновременно.

Решение

Сначала мы решаем для v0v0, используя x = x0 + v0t + 12at2: x = x0 + v0t + 12at2: x − x0 = v0t + 12at2x − x0 = v0t + 12at21.0 × 106m = v0 (120.0s) +12 (20,0 м / с2) (120,0 с) 21,0 × 106 м = v0 (120,0 с) +12 (20,0 м / с2) (120,0 с) 2v0 = 7133,3 м / с. V0 = 7133,3 м / с.

Затем мы подставляем v0v0 в v = v0 + atv = v0 + at, чтобы найти окончательную скорость:

v = v0 + at = 7133.3 м / с + (20,0 м / с2) (120,0 с) = 9533,3 м / с. V = v0 + at = 7133,3 м / с + (20,0 м / с2) (120,0 с) = 9533,3 м / с.

Значение

Есть шесть переменных смещения, времени, скорости и ускорения, которые описывают движение в одном измерении. Начальные условия данной задачи могут быть множеством комбинаций этих переменных. Из-за такого разнообразия решения могут быть не такими простыми, как простая подстановка в одно из уравнений. Этот пример показывает, что решения кинематики могут потребовать решения двух одновременных кинематических уравнений.

Освоив основы кинематики, мы можем перейти ко многим другим интересным примерам и приложениям. В процессе разработки кинематики мы также увидели общий подход к решению проблем, который дает как правильные ответы, так и понимание физических взаимоотношений. Следующий уровень сложности в наших задачах кинематики включает движение двух взаимосвязанных тел, называемых задачами преследования двух тел .

Задачи преследования двух тел

До этого момента мы рассматривали примеры движения с участием одного тела.Даже для задачи с двумя автомобилями и тормозным путем на мокрой и сухой дороге мы разделили эту задачу на две отдельные задачи, чтобы найти ответы. В задаче преследования двух тел движения объектов связаны, то есть искомое неизвестное зависит от движения обоих объектов. Чтобы решить эти проблемы, мы пишем уравнения движения для каждого объекта, а затем решаем их одновременно, чтобы найти неизвестное. Это показано на Рисунке 3.25.

Рис. 3.25 Сценарий преследования с двумя телами, в котором автомобиль 2 имеет постоянную скорость, а автомобиль 1 идет сзади с постоянным ускорением.Автомобиль 1 догонит автомобиль 2 позже.

Время и расстояние, необходимое для того, чтобы автомобиль 1 догнал автомобиль 2, зависит от начального расстояния, на которое автомобиль 1 находится от автомобиля 2, а также от скорости обоих автомобилей и ускорения автомобиля 1. Кинематические уравнения, описывающие движение обоих автомобилей, должны быть решил найти эти неизвестные.

Рассмотрим следующий пример.

Пример 3.13

Гепард ловит газель

Гепард прячется за кустом. Гепард замечает пробегающую мимо газель со скоростью 10 м / с.В момент, когда газель проезжает мимо гепарда, гепард ускоряется из состояния покоя со скоростью 4 м / с 2 , чтобы поймать газель. а) Сколько времени требуется гепарду, чтобы поймать газель? б) Что такое смещение газели и гепарда?

Стратегия

Мы используем систему уравнений для постоянного ускорения, чтобы решить эту проблему. Поскольку есть два движущихся объекта, у нас есть отдельные уравнения движения, описывающие каждое животное. Но то, что связывает уравнения, — это общий параметр, который имеет одинаковое значение для каждого животного.Если мы внимательно рассмотрим проблему, становится ясно, что общим параметром для каждого животного является их положение x , позднее t . Поскольку оба они начинаются с x0 = 0x0 = 0, их смещения будут такими же в более позднее время t , когда гепард догонит газель. Если мы выберем уравнение движения, которое решает смещение для каждого животного, мы можем затем установить уравнения, равные друг другу, и решить для неизвестного, то есть времени.

Решение
  1. Уравнение для газели: Газель имеет постоянную скорость, которая является ее средней скоростью, поскольку она не ускоряется.Поэтому мы используем уравнение 3.10 с x0 = 0x0 = 0:
    x = x0 + v – t = v – t. x = x0 + v – t = v – t.
    Уравнение для гепарда: гепард ускоряется из состояния покоя, поэтому мы используем уравнение 3.13 с x0 = 0x0 = 0 и v0 = 0v0 = 0: x = x0 + v0t + 12at2 = 12at2.x = x0 + v0t + 12at2 = 12at2.
    Теперь у нас есть уравнение движения для каждого животного с общим параметром, который можно исключить, чтобы найти решение. В этом случае мы решаем для t : x = v – t = 12at2t = 2v – a.x = v – t = 12at2t = 2v – a.
    Газель имеет постоянную скорость 10 м / с, что составляет ее среднюю скорость.Ускорение гепарда составляет 4 м / с 2 . Оценив т , время, за которое гепард достигнет газели, имеем
    t = 2v – a = 2 (10 м / с) 4m / s2 = 5s. t = 2v – a = 2 (10 м / с) 4m / s2 = 5s.
  2. Чтобы получить смещение, мы используем уравнение движения гепарда или газели, поскольку оба они должны дать одинаковый ответ.
    Смещение гепарда:
    x = 12at2 = 12 (4 м / с2) (5) 2 = 50 м. x = 12at2 = 12 (4 м / с2) (5) 2 = 50 м.
    Водоизмещение газели:
    x = v – t = 10 м / с (5) = 50 м. x = v – t = 10 м / с (5) = 50 м.Мы видим, что оба смещения равны, как и ожидалось.
Значение

Важно анализировать движение каждого объекта и использовать соответствующие кинематические уравнения для описания отдельного движения. Также важно иметь хорошую визуальную перспективу задачи преследования двух тел, чтобы увидеть общий параметр, который связывает движение обоих объектов.

Проверьте свое понимание 3.6

Велосипед имеет постоянную скорость 10 м / с. Человек начинает с отдыха и начинает бежать, чтобы догнать велосипед через 30 секунд, когда велосипед находится в том же положении, что и человек.Какое ускорение у человека?

Графический анализ одномерного движения

Цели обучения

К концу этого раздела вы сможете:

  • Опишите прямолинейный график с точки зрения его наклона и интервала y .
  • Определите среднюю или мгновенную скорость по графику положения в зависимости от времени.
  • Определите среднее или мгновенное ускорение по графику зависимости скорости отвремя.
  • Постройте график зависимости скорости от времени из графика положения от времени.
  • Постройте график зависимости ускорения от времени из графика зависимости скорости от времени.

График, как и картинка, стоит тысячи слов. Графики содержат не только числовую информацию; они также раскрывают отношения между физическими величинами. В этом разделе используются графики перемещения, скорости и ускорения в зависимости от времени, чтобы проиллюстрировать одномерную кинематику.

Склоны и общие отношения

Во-первых, обратите внимание, что графики в этом тексте имеют перпендикулярные оси, одна горизонтальная, а другая вертикальная.Когда две физические величины нанесены друг на друга на таком графике, горизонтальная ось обычно считается независимой переменной , а вертикальная ось — зависимой переменной . Если мы назовем горизонтальную ось x осью, а вертикальную ось y осью, как на рисунке 1, прямолинейный график будет иметь общий вид

.

[латекс] y = \ text {mx} + b \\ [/ latex]

Здесь м — это наклон , определяемый как подъем, разделенный на пробег (как показано на рисунке) прямой линии.Буква b используется для точки пересечения y , которая является точкой, в которой линия пересекает вертикальную ось.

График смещения от времени (

a = 0, поэтому v постоянно)

Время обычно является независимой переменной, от которой зависят другие величины, такие как смещение. График перемещения в зависимости от времени, таким образом, будет иметь x по вертикальной оси и t по горизонтальной оси.Рисунок 2 — это именно такой прямолинейный график. На нем показан график перемещения в зависимости от времени для реактивного автомобиля на очень плоском высохшем дне озера в Неваде.

Используя соотношение между зависимыми и независимыми переменными, мы видим, что наклон на графике выше — это средняя скорость [латекс] \ bar {v} \\ [/ latex], а точка пересечения — смещение в нулевой момент времени, то есть x 0 . Подставляя эти символы в [latex] y = \ text {mx} + b \\ [/ latex], получаем

[латекс] x = \ bar {v} t + {x} _ {0} \\ [/ latex]

или

[латекс] x = {x} _ {0} + \ bar {v} t \\ [/ latex].

Таким образом, график смещения в зависимости от времени дает общую взаимосвязь между смещением, скоростью и временем, а также дает подробную числовую информацию о конкретной ситуации.

Уклон x против т

Наклон графика перемещения x в зависимости от времени t — это скорость v .

[латекс] \ text {slope} = \ frac {\ Delta x} {\ Delta t} = v \\ [/ latex]

Обратите внимание, что это уравнение аналогично уравнению, полученному алгебраически из других уравнений движения в уравнениях движения для постоянного ускорения в одном измерении.

Из рисунка видно, что автомобиль имеет водоизмещение 400 м в момент времени 0,650 м при т, = 1,0 с и так далее. Его смещение в моменты времени, отличные от перечисленных в таблице, можно увидеть на графике; кроме того, информация о его скорости и ускорении также может быть получена из графика.

Пример 1. Определение средней скорости по графику смещения в зависимости от времени: Jet Car

Найдите среднюю скорость автомобиля, положение которого показано на рисунке 2.

Стратегия

Наклон графика x по сравнению с т. — это средняя скорость, поскольку наклон равен подъему за пробег. В этом случае подъем = изменение рабочего объема, а бег = изменение во времени, так что

[латекс] \ text {slope} = \ frac {\ Delta x} {\ Delta t} = \ bar {v} \\ [/ latex].

Поскольку наклон здесь постоянный, любые две точки на графике могут использоваться для определения наклона. (Вообще говоря, точнее всего использовать две широко разнесенные точки на прямой.Это связано с тем, что любая ошибка при чтении данных с графика пропорционально меньше, если интервал больше.)

Решение

1. Выберите две точки на линии. В этом случае мы выбираем точки, помеченные на графике: (6,4 с, 2000 м) и (0,50 с, 525 м). (Учтите, однако, что вы можете выбрать любые две точки.)

2. Подставьте значения x и t выбранных точек в уравнение. Помните, что при вычислении изменения (Δ) мы всегда используем конечное значение минус начальное значение.

[латекс] \ bar {v} = \ frac {\ Delta x} {\ Delta t} = \ frac {\ text {2000 m} — \ text {525 m}} {6 \ text {.} \ Text { 4 s} -0 \ text {.} \ Text {50 s}} \\ [/ latex],

дает

[латекс] \ bar {v} = \ text {250 м / с} [/ латекс].

Обсуждение

Это впечатляюще большая сухопутная скорость (900 км / ч или около 560 миль / ч): намного больше, чем типичное ограничение скорости на шоссе в 60 миль / ч (27 м / с или 96 км / ч), но значительно скромнее. рекорда 343 м / с (1234 км / ч или 766 миль / ч), установленного в 1997 году.

Графики движения, когда

a постоянна, но a ≠ 0

Графики на Рисунке 3 ниже представляют движение автомобиля с реактивным двигателем, когда он набирает максимальную скорость, но только в то время, когда его ускорение является постоянным. Время для этого движения начинается с нуля (как если бы оно измерялось секундомером), а смещение и скорость изначально составляют 200 м и 15 м / с соответственно.

График смещения в зависимости от времени на рис. 3 (а) представляет собой кривую, а не прямую линию.Наклон кривой становится более крутым с течением времени, показывая, что скорость увеличивается с течением времени. Наклон в любой точке графика зависимости смещения от времени — это мгновенная скорость в этой точке. Его можно найти, проведя прямую касательную к кривой в интересующей точке и взяв наклон этой прямой. Касательные линии показаны для двух точек на рисунке 3 (а). Если это делается в каждой точке кривой и значения наносятся в зависимости от времени, то получается график зависимости скорости от времени, показанный на рисунке 3 (b).Кроме того, наклон графика зависимости скорости от времени — это ускорение, которое показано на Рисунке 3 (c).

Пример 2. Определение мгновенной скорости по уклону в точке: Реактивный автомобиль

Рассчитайте скорость реактивного автомобиля за время 25 с, найдя наклон графика x против t на графике ниже.

Наклон графика x против т — это скорость. Это показано в двух точках. Мгновенная скорость в любой точке — это наклон касательной в этой точке.

Стратегия

Наклон кривой в точке равен наклону прямой, касательной к кривой в этой точке. Этот принцип проиллюстрирован на рисунке 5, где Q — точка при t = 25 с.

Решение

1. Найдите касательную к кривой при t = 25 с.

2. Определите конечные точки касательной. Они соответствуют положению 1300 м за 19 с и положению 3120 м за 32 с.

3. Подставьте эти конечные точки в уравнение для определения наклона: v .

[латекс] \ text {slope} = {v} _ {Q} = \ frac {{\ Delta x} _ {Q}} {{\ Delta t} _ {Q}} = \ frac {\ left (\ text {3120 m} — \ text {1300 m} \ right)} {\ left (\ text {32 s} — \ text {19 s} \ right)} \\ [/ latex]

Таким образом,

[латекс] {v} _ {Q} = \ frac {\ text {1820 m}} {\ text {13 s}} = \ text {140 м / с.} \\ [/ latex]

Обсуждение

Это значение, приведенное в таблице этого рисунка для v при t = 25 с.Значение 140 м / с для v Q показано на рисунке 5. Таким образом можно получить весь график v и t .

Продолжая этот шаг дальше, отметим, что наклон графика зависимости скорости от времени — это ускорение. Склон делится на подъем, разделенный бегом; на графике v и t , подъем = изменение скорости Δ v и бег = изменение во времени Δ t .

Наклон v vs. т

Наклон графика зависимости скорости v от времени t — это ускорение a .

[латекс] \ text {slope} = \ frac {\ Delta v} {\ Delta t} = a \\ [/ latex]

Поскольку график зависимости скорости от времени на рисунке 3 (b) представляет собой прямую линию, ее наклон везде одинаковый, что означает, что ускорение является постоянным. Ускорение в зависимости от времени показано на рисунке 3 (c).

Дополнительную общую информацию можно получить из рисунка 5 и выражения для прямой линии [латекс] y = \ text {mx} + b \\ [/ latex].

В этом случае вертикальная ось y составляет V , точка пересечения b составляет v 0 , наклон м составляет a , а горизонтальная ось x составляет t . Подставляя эти символы, получаем

[латекс] v = {v} _ {0} + \ text {at} \\ [/ latex].

Общая зависимость скорости, ускорения и времени снова была получена из графика. Обратите внимание, что это уравнение также было получено алгебраически из других уравнений движения в уравнениях движения для постоянного ускорения в одном измерении.

Неслучайно те же уравнения получаются графическим анализом и алгебраическими методами. Фактически, важный способ обнаружения физических взаимосвязей — это измерение различных физических величин, а затем построение графиков одной величины относительно другой, чтобы увидеть, коррелированы ли они каким-либо образом. Корреляции подразумевают физические отношения и могут быть показаны в виде гладких графиков, подобных приведенным выше. Из таких графиков иногда можно постулировать математические отношения.Затем проводятся дальнейшие эксперименты для определения достоверности предполагаемых соотношений.

Графики движения при непостоянном ускорении

Теперь рассмотрим движение реактивного автомобиля от 165 м / с до максимальной скорости 250 м / с, как показано на рисунке 6. Время снова начинается с нуля, а начальное смещение и скорость составляют 2900 м и 165 м. / с соответственно. (Это были окончательное смещение и скорость автомобиля в движении, показанные на Рисунке 3.) Ускорение постепенно уменьшается от 5,0 м / с 2 до нуля при наезде на 250 м / с. Наклон графика x против t увеличивается до t = 55 с, после чего наклон остается постоянным. Точно так же скорость увеличивается до 55 с, а затем становится постоянной, поскольку ускорение уменьшается до нуля на 55 с и остается нулевым после этого.

Пример 3. Расчет ускорения по графику зависимости скорости от времени

Рассчитайте ускорение реактивного автомобиля за время 25 с, найдя наклон v vs. t график на рисунке 6 (б).

Стратегия

Наклон кривой при t = 25 с равен наклону касательной прямой в этой точке, как показано на рисунке 6 (b).

Решение

Определите конечные точки касательной по рисунку, а затем подставьте их в уравнение для определения наклона, a .

[латекс] \ text {slope} = \ frac {\ Delta v} {\ Delta t} = \ frac {\ left (\ text {260 м / с} — \ text {210 м / с} \ right)} {\ left (\ text {51 s} -1.{2} \\ [/ латекс].

Обсуждение

Обратите внимание, что это значение для a согласуется со значением, приведенным на рисунке 6 (c) при t = 25 с.

График перемещения в зависимости от времени можно использовать для построения графика зависимости скорости от времени, а график зависимости скорости от времени можно использовать для построения графика ускорения в зависимости от времени. Мы делаем это, находя наклон графиков в каждой точке. Если график линейный (то есть линия с постоянным наклоном), легко найти наклон в любой точке, и у вас есть наклон для каждой точки.Графический анализ движения может использоваться для описания как частных, так и общих характеристик кинематики. Графики также можно использовать для других тем по физике. Важным аспектом изучения физических отношений является их графическое отображение и поиск лежащих в основе отношений.

Проверьте свое понимание

График зависимости скорости от времени захода корабля в гавань показан ниже. (а) Опишите движение корабля на основе графика. б) Как будет выглядеть график ускорения корабля?

Решение

(a) Корабль движется с постоянной скоростью, а затем начинает замедляться с постоянной скоростью.В какой-то момент скорость его замедления снижается. Он поддерживает эту более низкую скорость замедления, пока не перестанет двигаться.

(b) График ускорения в зависимости от времени покажет нулевое ускорение на первом отрезке, большое и постоянное отрицательное ускорение на втором отрезке и постоянное отрицательное ускорение.

Сводка раздела

  • Графики движения можно использовать для анализа движения.
  • Графические решения дают идентичные решения математическим методам вывода уравнений движения.
  • Наклон графика смещения x в зависимости от времени t — это скорость v .
  • Наклон графика зависимости скорости v от t — это ускорение a .
  • Средняя скорость, мгновенная скорость и ускорение могут быть получены путем анализа графиков.

Концептуальные вопросы

1. (a) Объясните, как можно использовать график зависимости положения от времени на рисунке 9 для описания изменения скорости во времени.Определите: (b) время ( t a , t b , t c , t d или t e ), при котором мгновенная скорость равна наибольшее, (c) время, в которое оно равно нулю, и (d) время, когда оно отрицательно.

Рисунок 9.

2. (a) Изобразите график зависимости скорости от времени, соответствующий графику перемещения от времени, представленному на рисунке 10. (b) Определите время или время ( t a , t b , т в и т. д.), при которой мгновенная скорость максимальна. (c) В какое время он равен нулю? (г) В какое время он отрицательный?

3. (a) Объясните, как можно определить ускорение с течением времени на основе графика зависимости скорости от времени, такого как на рисунке 11. (b) На основании графика, как ускорение изменяется с течением времени?

4. (a) Изобразите график зависимости ускорения от времени, соответствующий графику зависимости скорости от времени, представленному на рисунке 12. (b) Определите время или времена ( t a , t b , т в и т. д.), при котором ускорение наибольшее. (c) В какое время он равен нулю? (г) В какое время он отрицательный?

Рисунок 12.

5. Рассмотрим график зависимости скорости от времени человека в лифте, показанный на рисунке 13. Предположим, лифт изначально находится в состоянии покоя. Затем он ускоряется в течение 3 секунд, поддерживает эту скорость в течение 15 секунд, затем замедляется на 5 секунд, пока не остановится. Ускорение для всей поездки не является постоянным, поэтому мы не можем использовать уравнения движения из уравнений движения для постоянного ускорения в одном измерении для всей поездки.(Однако мы могли бы использовать их в трех отдельных разделах, где ускорение является константой.) Нарисуйте графики: (а) положения в зависимости от времени и (б) ускорения в зависимости от времени для этой поездки.

6. Цилиндр толкается, а затем скатывается по наклонной плоскости. Если начало координат является начальной точкой, нарисуйте положение, скорость и ускорение цилиндра в зависимости от времени, когда он поднимается, а затем опускается по плоскости.

Задачи и упражнения

Примечание: всегда есть неопределенность в числах, взятых из графиков.Если ваши ответы отличаются от ожидаемых значений, проверьте их, чтобы увидеть, находятся ли они в пределах оцененных вами неопределенностей извлечения данных.

1. a) Взяв наклон кривой на рисунке 14, убедитесь, что скорость реактивного вагона составляет 115 м / с при t = 20 с. (b) Взяв наклон кривой в любой точке на Рисунке 15, убедитесь, что ускорение реактивного автомобиля составляет 5,0 м / с 2 .

2. Используя приблизительные значения, рассчитайте наклон кривой на рисунке 16, чтобы убедиться, что скорость при t = 10.0 с составляет 0,208 м / с. Предположим, что все значения известны с 3 значащими цифрами.

3. Используя приблизительные значения, рассчитайте наклон кривой на Рисунке 16, чтобы убедиться, что скорость при t = 30,0 с составляет 0,238 м / с. Предположим, что все значения известны с 3 значащими цифрами.

4. Взяв наклон кривой на рисунке 17, убедитесь, что ускорение составляет 3,2 м / с 2 при t = 10 с.

Рисунок 17.

5. Постройте график смещения для маршрутного поезда метро, ​​как показано на рисунке 2 из ускорения (снова показано ниже).Ваш график должен показывать положение поезда в километрах от t = 0 до 20 с. Вам нужно будет использовать информацию об ускорении и скорости, приведенную в примерах к этому рисунку.

Поезд метро в Сан-Паулу, Бразилия, сбавляет скорость при входе на станцию. Он ускоряется в направлении, противоположном направлению его движения. (Источник: Юсуке Кавасаки, Flickr)

6. (a) Взяв наклон кривой на рисунке 11, найдите скорость бегуна при t = 2.5 с. (b) Повторить через 7,5 с. Эти значения должны соответствовать графику на Рисунке 18.

7. График v ( t ) показан для бегового спринтера мирового класса в беге на 100 м. (См. Рисунок 21). а) Какова его средняя скорость в первые 4 с? (b) Какова его мгновенная скорость при t = 5 с? (c) Каково его среднее ускорение между 0 и 4 с? (г) Сколько у него времени для забега?

8. На рисунке 22 показан график смещения частицы за 5 с.Нарисуйте соответствующие графики скорости и ускорения.

независимая переменная:
переменная, относительно которой измеряется зависимая переменная; обычно наносится по оси x
зависимая переменная:
переменная, которая измеряется; обычно наносится по оси y
наклон:
разница в y -значение (подъем), деленная на разницу в x -значение (пробег) двух точек на прямой
Y-перехват:
значение y , когда x = 0, или когда график пересекает ось y

Избранные решения проблем и упражнения

1.{3} \ text {m}} {\ left (40 \ text {.} \ Text {0–20} .0 \ right) \ text {s}} = \ text {238 м / с} \\ [/ латекс]

Рисунок 23.

5.

7. (а) 6 м / с (б) 12 м / с (в) 3 м / с 2 (г) 10 с

Графическое изображение движения — Физика тела: движение к метаболизму

Графики движения — полезный инструмент для визуализации и передачи информации о движении объекта. Наша цель — создать графики движения для нашего примера парашютиста, но сначала давайте удостоверимся, что мы поняли основную идею.

Мы начнем с рассмотрения графиков движения объекта с начальным положением 2 м и постоянной скоростью 4 м / с . Объект, движущийся с постоянной скоростью, имеет нулевое ускорение, поэтому график зависимости ускорения от времени остается нулевым:

График зависимости ускорения от времени для объекта с постоянной скоростью равен нулю.

Скорость постоянна, поэтому на графике зависимости скорости от времени останется значение 4 м / с. :

.
Скорость vs.график времени плоский (постоянный) на отметке 4 м / с .

Скорость — это скорость, с которой изменяется положение, поэтому график зависимости положения от времени должен изменяться с постоянной скоростью, начиная с исходного положения (в нашем примере 2 м ). Наклон графика движения сообщает нам скорость изменения переменной на вертикальной оси, поэтому мы можем понимать скорость как наклон графика положения в зависимости от времени.

График зависимости положения от времени является линейным с наклоном, равным скорости 4 м / с и точкой пересечения, равным начальному положению 2 м .График пересекает позицию 10 метров за время 2 секунды.

Теперь давайте посмотрим на графики движения объекта с постоянным ускорением. Давайте дадим нашему объекту то же начальное положение 2 м / и начальную скорость 4 м / с , а теперь постоянное ускорение 2 м / с . Ускорение в зависимости от времени остается постоянным и составляет 2 м / с / с :

.
График зависимости ускорения от времени является плоским при значении ускорения, в этом примере 2 м / с / с

Ускорение — это скорость, с которой изменяется скорость, поэтому ускорение — это наклон зависимости скорости отграфик времени. Для нашего постоянного ускорения 2 м / с / с график скорости должен иметь постоянный наклон 2 м / с / с :

График зависимости скорости от времени является линейным с наклоном, равным значению ускорения 2 м / с и точкой пересечения, равным начальному значению скорости 4 м / с .

Наконец, если скорость изменяется с постоянной скоростью, то наклон графика положения, который представляет скорость, также должен изменяться с постоянной скоростью. Результатом изменения наклона является изогнутый график, а именно кривая с постоянно изменяющимся наклоном — это параболическая кривая или парабола .

График положения и времени объекта с постоянным ускорением представляет собой параболическую кривую. Кривизна направлена ​​вверх для положительного ускорения и вниз для отрицательного ускорения. Перехват — это начальная позиция, в этом примере 2 м .

Мы не построили графики движения для ситуации постоянного положения, потому что они относительно неинтересны. График положения постоянен при начальном значении положения, график скорости постоянен при нуле, а график ускорения также постоянен при нуле.Давайте закончим этот раздел некоторыми интересными графиками — графиками объекта, который меняет направление. Например, объект, подброшенный в воздух с начальной скоростью 5 м / с , из начального положения 2 м , который затем падает на землю на высоте 0 м . Пренебрегая сопротивлением воздуха, ускорение будет постоянным при отрицательных g, или -9,8 м / с / с .

График зависимости ускорения от времени для объекта плоский и составляет -9,8 м / с / с (для выбора направления вниз в качестве отрицательного направления).

Скорость будет положительной, но с замедлением до нуля, пересечет нулевое значение по мере поворота объекта, а затем начнет увеличиваться в отрицательном направлении.

График зависимости скорости от времени начинается с отметки 5 м / с и линейно уменьшается, пересекая нулевое значение примерно на 0,5 с , а затем становится более отрицательным со временем линейно и достигает — 5 м / с сразу после 1 с. . Уклон -9,8 м / с / .

Положение будет увеличиваться по мере движения объекта вверх, затем уменьшаться по мере его падения вниз параболическим образом, потому что наклон изменяется с постоянной скоростью (ускорение постоянно, поэтому скорость изменяется с постоянной скоростью, поэтому наклон положения график постоянно меняется).

График зависимости положения от времени представляет собой параболу с нисходящей кривизной, начинающейся на высоте 2 м , с пиком около 3,3 м примерно на 0,5 с , проходящей обратно через 2 м сразу после 1 с и ударяющейся о землю. сразу после 1,3 с .

Ознакомьтесь с этой интерактивной симуляцией движущегося человека и соответствующими графиками движения:

Повседневный пример: конечная скорость

Давайте посмотрим на графики движения нашего парашютиста, когда он достигает предельной скорости -120 миль в час , что составляет около 54 м / с .Давайте установим нашу начальную позицию для этого анализа как позицию, в которой они достигают предельной скорости.

Ускорение равно нулю, потому что они достигли предельной скорости:

График зависимости ускорения от времени постоянный (плоский) на нуле.

Скорость постоянная, но отрицательная:

График зависимости скорости от времени постоянен около -52 м / с .

И позиция изменяется с постоянной скоростью, со временем становясь все более отрицательной.

График зависимости положения от времени линейно уменьшается от нуля до -520 м через 10 с .

Пример на каждый день: полный прыжок с парашютом

Теперь давайте посмотрим на графики движения нашего парашютиста до достижения конечной скорости, начиная с начального прыжка.

График зависимости ускорения от времени начинается с -9,8 м / с / с , потому что в первый момент нет силы сопротивления, поэтому дайвер на мгновение находится в свободном падении. По мере набора скорости сила лобового сопротивления увеличивается, компенсируя большую часть веса, поэтому ускорение стремится к нулю и становится неотличимым от нуля около 15 с .

График зависимости скорости от времени начинается с нуля и потому, что начальная скорость была нулевой. Скорость остается отрицательной, потому что движение идет вниз, но наклон не постоянный, как при свободном падении, потому что ускорение не является постоянным, как при свободном падении. Это потому, что сила сопротивления растет с увеличением скорости, в конечном итоге становится такой же большой, как вес, поэтому скорость в конечном итоге начинает выравниваться и приближаться к постоянной 52 м / с .

График зависимости положения от времени начинается на отметке 3660 м и уменьшается к нулю с отрицательным и постепенно увеличивающимся наклоном (движение вниз и ускорение).После 20 с парашютист приближается к позиции 2750 м , и наклон становится постоянным на уровне 52 м / с , , что указывает на конечную скорость. Обратите внимание, что мы преобразовали нашу [pb_glossary] начальную позицию [/ pb_glossary] 12 000 футов в эквивалентную 3660 м .

Веб-сайт класса физики

Круговое движение и гравитация: обзор набора задач

Этот набор из 27 задач нацелен на вашу способность комбинировать законы Ньютона и уравнения кругового движения и гравитации для анализа движения объектов, движущихся по кругу, включая орбитальные спутники.Проблемы варьируются по сложности от очень простых и простых до очень сложных и сложных. Более сложные задачи имеют цветовую кодировку , синие задачи .

Характеристики движения объектов, движущихся по кругам.

Объекты, движущиеся по кругу, имеют скорость, равную пройденному за время пути расстоянию. Расстояние вокруг круга эквивалентно длине окружности и рассчитывается как 2 • pi • R, где R — радиус.Время одного оборота по окружности называется периодом и обозначается символом T. Таким образом, средняя скорость объекта, движущегося по кругу, определяется выражением 2 • pi • R / T. Часто в постановке задачи указывается частота вращения в оборотах в минуту или в оборотах в секунду. Каждый оборот по окружности эквивалентен длине окружности. Таким образом, умножение частоты вращения на длину окружности позволяет определить среднюю скорость объекта.

Ускорение объектов, движущихся по кругу, основано в первую очередь на изменении направления. Фактическая скорость ускорения зависит от скорости изменения направления и напрямую связана со скоростью и обратно пропорциональна радиусу поворота. В итоге ускорение определяется выражением v 2 / R, где v — скорость, а R — радиус окружности.

Уравнения для средней скорости (v) и среднего ускорения (a) приведены ниже.

v = d / t = 2 • pi • R / T = частота • 2 • pi • R
а = v 2 / R

Направленные величины для объектов, движущихся по кругам

Успешный математический анализ объектов, движущихся по кругу, во многом зависит от концептуального понимания направления векторов ускорения и результирующей силы. Движение по круговой траектории требует чистой силы, направленной к центру круга.В каждой точке пути результирующая сила должна быть направлена ​​внутрь. Хотя может существовать отдельная сила, направленная наружу, должна существовать внутренняя сила, которая подавляет ее по величине и удовлетворяет требованию для внутренней чистой силы. Поскольку чистая сила и ускорение всегда в одном и том же направлении, ускорение объектов, движущихся по кругу, также должно быть направлено внутрь.

Диаграммы свободного тела и второй закон Ньютона

Часто силовой анализ должен проводиться для объекта, движущегося по кругу.Целью анализа является определение величины отдельной силы, действующей на объект, или использование значений отдельных сил для определения ускорения. Как и любая задача анализа сил, эти задачи должны начинаться с построения диаграммы свободного тела, показывающей тип и направление всех сил, действующих на объект. Из диаграммы F net = m • можно записать уравнение. При написании уравнения помните, что сетка F представляет собой векторную сумму всех индивидуальных сил.Лучше всего это записать, сложив все силы, действующие в направлении ускорения (внутрь), и вычтя те, которые ему противостоят. Два примера показаны на рисунке ниже.

Закон всемирного тяготения Ньютона

Спутники, движущиеся по орбите, — это просто снаряды — объекты, на которые действует только сила тяжести. Сила, управляющая их движением, — это сила гравитационного притяжения к объекту, который находится в центре их орбиты.Планеты вращаются вокруг Солнца в результате гравитационной силы притяжения к Солнцу. Естественные луны вращаются вокруг планет в результате гравитационной силы притяжения к планете. Гравитация — это сила, которая действует на больших расстояниях таким образом, что любые два объекта с массой будут притягиваться. Ньютон был первым, кто предложил теорию, чтобы описать это универсальное массовое притяжение и выразить его математически. Закон, известный как закон всемирного тяготения, гласит, что сила гравитационного притяжения прямо пропорциональна произведению масс и обратно пропорциональна квадрату расстояния между их центрами.В форме уравнения,

F grav = G • m 1 • m 2 / d 2

где m 1 и m 2 — массы притягивающих объектов (в кг), d — расстояние разделения, измеренное от центра объекта к центру объекта (в метрах), а G — константа пропорциональности (иногда называемая всемирная гравитационная постоянная). Значение G составляет 6,673 x 10 -11 Н • м 2 / кг 2 .

Ускорение свободного падения

Поскольку на орбитальные спутники действует исключительно сила тяжести, их ускорение является ускорением силы тяжести (g). На земной поверхности это значение составило 9,8 м / с 2 . Для местоположений, отличных от поверхности Земли, необходимо уравнение, которое выражает g через соответствующие переменные. Ускорение свободного падения зависит от массы объекта, который находится в центре орбиты (M central ), и расстояния разделения от этого объекта (d).Уравнение, связывающее эти две переменные с ускорением свободного падения, получено из закона всемирного тяготения Ньютона. Уравнение

g = G • M центральный / d 2

где G составляет 6,673 x 10 -11 Н • м 2 / кг 2 .

Орбитальная скорость

Скорость, необходимая спутнику, чтобы оставаться на орбите вокруг центрального тела (планеты, солнца, другой звезды и т. Д.).) зависит от радиуса орбиты и массы центрального тела. Уравнение, выражающее взаимосвязь между этими переменными, получается путем объединения определений ускорения кругового движения с законом всемирного тяготения Ньютона. Уравнение

v = SQRT (G • M центральный / R)

где M central — масса центрального тела, вокруг которого вращается спутник, R — радиус орбиты, а G — 6,673 x 10 -11 Н • м 2 / кг 2 .

Орбитальный период

Для общего движения объекта по кругу период связан с радиусом круга и скоростью объекта уравнением v = 2 • pi • R / T. В случае орбитального спутника это уравнение для скорости можно приравнять к уравнению для орбитальной скорости, полученной из всемирного тяготения, чтобы получить новое уравнение для орбитального периода. Результат вывода:

T 2 / R 3 = 4 • pi 2 / (G • M центральный )

где M central — масса центрального тела, вокруг которого вращается спутник, R — радиус орбиты, а G — 6.673 x 10 -11 Н • м 2 / кг 2 . Выраженное таким образом уравнение показывает, что отношение квадрата периода к радиусу в кубе для любого спутника, вращающегося вокруг центрального тела, одинаково независимо от природы спутника или радиуса его орбиты. Это соотношение зависит только от массы объекта, который втягивает орбитальный спутник внутрь. Этот принцип согласуется с третьим законом движения планет Кеплера.

Резюме математических формул

Одна из трудностей, с которыми студент может столкнуться в этом наборе задач, — это путаница относительно того, какую формулу использовать.В таблице ниже представлено полезное резюме формул, относящихся к круговому движению и движению спутника. В таблице многие формулы получены из других уравнений. Таким образом, часто будет несколько способов определения неизвестной величины. Подходя к этим проблемам, рекомендуется практиковать обычные привычки эффективного решателя проблем; определить известные и неизвестные величины в виде символов физических формул, разработать стратегию использования известных для решения неизвестного, а затем, наконец, выполнить необходимые алгебраические шаги и замены, необходимые для решения.

Для расчета … … используйте уравнение (а):
Скорость
(v)
v = 2 • pi • R / T
v = SQRT (G • M центральный / R) только для спутников
Разгон
(а)
a = v 2 / R или a = F net / m
a = g = G • M центральный / d 2 только для спутников
Чистая сила
(F net )
F net = m • a или F net = m • v 2 / R
F net = F grav = G • m sat • M центральный / d 2 только для спутников
Период
(Т)
T = 2 • pi • R / v
T 2 = 4 • pi 2 / (G • M центральный ) • R 3 только для спутников

Привычки эффективно решать проблемы

Эффективный решатель проблем по привычке подходит к физическим проблемам таким образом, чтобы отражать набор дисциплинированных привычек.Хотя не все эффективные специалисты по решению проблем используют один и тот же подход, все они имеют общие привычки. Эти привычки кратко описаны здесь. Эффективное решение проблем …

  • … внимательно читает задачу и создает мысленную картину физической ситуации. При необходимости они набрасывают простую схему физической ситуации, чтобы помочь визуализировать ее.
  • … определяет известные и неизвестные величины в организованном порядке, часто записывая их на диаграмме.Они приравнивают заданные значения к символам, используемым для представления соответствующей величины (например, m = 61,7 кг, v = 18,5 м / с, R = 30,9 м, F norm = ???).
  • … строит стратегию решения неизвестной величины; стратегия, как правило, сосредоточена вокруг использования физических уравнений и во многом зависит от понимания физических принципов.
  • … определяет подходящую (ые) формулу (ы) для использования, часто записывая их. При необходимости они выполняют необходимое преобразование количеств в правильные единицы.
  • … выполняет подстановки и алгебраические манипуляции, чтобы найти неизвестную величину.

Подробнее …

Дополнительная литература / Учебные пособия:

Следующие страницы учебного пособия по физике могут быть полезны для понимания концепций и математики, связанных с этими проблемами.

Набор задач кругового движения и гравитации

Просмотреть набор задач

Решения с аудиогидом для кругового движения и гравитации

Просмотрите решение проблемы с аудиогидом:
1 | 2 | 3 | 4 | 5 | 6 | 7 | 8 | 9 | 10 | 11 | 12 | 13 | 14 | 15 | 16 | 17 | 18 | 19 | 20 | 21 | 22 | 23 | 24 | 25 | 26 | 27

Лаборатория 5 — Равномерное круговое движение

Введение

Если вы когда-либо были на аттракционе в парке развлечений, который проходит по изогнутой или круговой траектории, то вы испытали силу, называемую центростремительной силой, толкающую вас в аттракцион.Будь то задняя стенка «Раундапа» или «Ротора», аттракцион, где пол падает из-под ваших ног, или ремень безопасности «американских горок», создающих силу, вы постоянно ускоряетесь к центру аттракциона. кривизны. Если — чего вы больше всего боитесь в такой поездке — эта сила внезапно исчезнет, ​​вы двинетесь в направлении, касательном к круговой траектории. Вот что происходит, когда вы едете с холма на американских горках прямо перед тем, как ремень безопасности сработает.

На рисунке 1 сравнивается движение в присутствии центростремительной силы с результирующим движением тела, если центростремительная сила внезапно прекратится.

Рисунок 1 : Объект в круговом движении

В качестве другого примера рассмотрим шар, прикрепленный к веревке и вращающийся по кругу, как показано на рис. 2а. Натяжение струны прикладывает к шару центростремительную силу , заставляя его двигаться по круговой траектории. Струна тянет мяч к центру круга, в то время как мяч тянет наружу на струне и, следовательно, на вашей руке в соответствии с третьим законом действия и противодействия Ньютона.Таким образом, эта внешняя сила не действует на мяч, хотя ее часто и неправильно называют центробежной силой, действующей на мяч. Когда центростремительная сила прекращается, например, на рис. 2b, когда струна разрывается, объект движется в направлении скорости в этот момент. Это направление является касательным к окружности в этой точке.

Рисунок 2 : Вид сверху шара на тетиве до и после разрыва струны

Центростремительная сила, удерживающая вас в поездке, может быть определена с помощью нескольких измерений и расчетов.В этом эксперименте вы определите, какие переменные должны быть известны для определения центростремительной силы, необходимой для поддержания движения массы по круговой траектории с постоянной скоростью.

Обсуждение принципов

Второй закон Ньютона гласит, что результирующая сила

F net

, приложенная к движущемуся телу, равна произведению массы тела

m

и его ускорения

a.

(1)

F net = ma

При применении второго закона Ньютона к круговому движению удобно использовать координаты, параллельные и перпендикулярные движению объекта.У тела, движущегося по прямой линии, ускорение происходит из-за изменения величины скорости. Для тела, движущегося по круговой траектории с постоянной скоростью, величина | v | скорости не меняется, но направление вектора скорости v постоянно меняется. В этом случае объект не имеет ускорения в направлении, параллельном его движению, и

F ||

в уравнении. (2)

F || = ma ||

равно нулю.Это движение называется равномерным круговым движением — движением по круговой траектории с постоянной скоростью. Так как вектор скорости изменяется во времени, объект при равномерном круговом движении ускоряется. Концептуально использование параллельных и перпендикулярных координат удобно, потому что параллельная сила отвечает за изменения скорости, а перпендикулярная сила (или центростремительная сила) отвечает за изменения направления.

Центростремительное ускорение

Какое ускорение необходимо, чтобы тело двигалось по кругу с постоянной скоростью? См. Рис.3а ниже. В определенный момент частица оказывается на вершине радиус-вектора r. В другой момент времени

Δt

позже частица оказывается на вершине радиус-вектора r ‘. На малом интервале времени

Δt

тело движется по дуге длиной

vΔt

. Если

Δt

достаточно мало, длина дуги приблизительно равна расстоянию между положением тела в начале интервала

Δt

и положением в конце интервала.Этот отрезок прямой вместе с двумя радиусами образует равнобедренный треугольник, показанный на рис. 3а.

Рисунок 3 : Геометрические характеристики

На рис. 3b показаны векторы скорости в двух положениях. Обратите внимание, что величина вектора скорости не меняется. На рис. 3c два вектора скорости перерисованы без изменения их длины или ориентации. Стрелка, соединяющая концы векторов скорости, представляет изменение

Δv

скорости частицы из-за изменения направления в течение временного интервала

Δt

.Обратите внимание, что по мере уменьшения

Δt

угол между направлением

Δv

и

v

и

v ‘

приближается к прямому углу 90 °.

Поскольку касательная к окружности перпендикулярна ее радиусу, отсюда следует, что угол

θ

между

v

и

v ‘

, как показано на рис. 3c, совпадает с углом между двумя радиусами на рис. 3а. Из тригонометрии мы знаем, что эти два треугольника конгруэнтны, и поэтому отношения их сторон должны быть равны.Итак, мы можем написать следующее выражение

Разделив обе стороны на

Δt / v

, получим

Конечно,

Δv / Δt

— это ускорение тела, и оно находится в направлении

Δv

. Из рис. 3в видно, что ускорение направлено к центру окружности. Величина центростремительного ускорения

a c

определяется выражением

а центростремительная сила

Поскольку напрямую измерить скорость тела сложно, вместо этого вы будете вычислять скорость из величин, которые легче измерить.Величину вектора скорости можно определить путем измерения расстояния, которое объект проходит за единицу времени. Если

T

— это период (время, необходимое для того, чтобы объект совершил один полный оборот), то скорость равна расстоянию, пройденному за этот один оборот, деленному на период.

Подставляя это в формулу. (6) для центростремительного ускорения дает

(9)

a c = = 4 π 2 f 2 r = 4 π 2 T −2 r

где

f = 1 / T

— количество оборотов в секунду, измеренное в герцах.Теперь уравнение. (7) можно записать в терминах измеряемых величин

m

,

T

и

r

как

Цель

В этом эксперименте вы измеряете период вращения объекта и вычисляете центростремительную силу, действующую на него. Вы сравните эту центростремительную силу с эквивалентной силой, необходимой для поддержания объекта в том же радиусе.

Оборудование

  • Аппарат кругового движения

  • Ассорти по весу

  • Пузырьковый уровень

  • Остаток средств

  • Секундомер

  • Метрическая палка

  • Весна

Процедура

Хотя было бы интереснее провести это исследование на полпути, простой лабораторный прибор, показанный на рис.4 ниже будет использоваться для изучения природы центростремительной силы. Используя прибор центростремительной силы, вы можете измерить частоту вращения объекта, движущегося по круговой траектории с известным радиусом. Уравнение (10)

F c = 4 π 2 mf 2 r

затем можно использовать для вычисления центростремительной силы, действующей на объект.

Описание аппарата

На рисунке 4 показано устройство и его различные компоненты. Нижняя часть основания снабжена регулируемыми винтами, которые можно использовать для выравнивания всего устройства.Ригель с противовесом на конце можно перемещать для изменения положения вращающейся массы. Указатель можно перемещать по прорези и размещать прямо под кончиком вращающейся массы. Это помогает измерить радиус круговой траектории вращающейся массы.

Рисунок 4 : Эскиз, показывающий компоненты устройства

В этом аппарате центростремительная сила обеспечивается пружиной. Когда пружина прикреплена к вращающейся массе

M R

, она втягивается, как показано на рис.5а и 6б. Чтобы вернуть вращающуюся массу обратно над указателем или индикатором радиуса, струна пропускается через шкив и на конце струны добавляется достаточно массы, чтобы вернуть вращающуюся массу выше индикатора радиуса.

Рисунок 5 : Эскиз первоначальной настройки

Рисунок 6 : Фотография первоначальной настройки

На Рис. 5b и Рис. 6c вес

м h г

подвешенной массы

м h

просто уравновешивает упругую силу пружины, когда вращающийся объект

M R

находится над указателем или индикаторный столб радиуса.На фиг. 7 пружина растягивается на ту же величину, что и масса

M R

вращается по траектории с тем же радиусом, что и на фиг. 5b. Величина двух сил

F c

и

m h g

должна быть в близком соответствии, поскольку обе силы производят одинаковое удлинение пружины.

Рисунок 7 : Эскиз, показывающий круговой путь объекта

Процедура A: Измерение периода вращения

1

Выровняйте устройство с помощью трех регулировочных винтов в основании.Устройство находится в горизонтальном положении, когда перекладина остается зафиксированной в любом положении. При необходимости используйте пузырьковый уровень, чтобы помочь вам в этом процессе.

2

Взвесьте вращающуюся массу

M R

и введите это значение в рабочий лист.

3

Установите индикатор радиуса для наименьшего радиуса и измерьте

r

. Радиус измеряется от центра стойки (оси вращения) до центра индикатора радиуса.

Запишите

r

в таблице данных на рабочем листе.

4

Перемещайте перекладину, пока кончик вращающейся массы не окажется над индикатором радиуса.

5

Присоедините пружину к вращающейся массе.

Присоедините струну к вращающейся массе, протяните струну над шкивом и прикрепите ее к подвеске массы на другом конце.

Добавляйте гири к подвеске, пока кончик вращающейся гири не окажется над индикатором радиуса.

Запишите значение общей подвешенной массы

м ч

в таблице данных 1 на рабочем листе.

6

Снимите массы и подвеску и поверните перекладину.Убедитесь, что кончик вращающейся массы ударяет по индикатору радиуса каждый оборот. Возможно, вам придется немного попрактиковаться, прежде чем начать сбор данных. Важно равномерно вращать вал.

ВНИМАНИЕ! Следите за тем, чтобы волосы и одежда не попадали на устройство, когда оно движется!

С помощью таймера запишите время, необходимое для 50 оборотов. Запишите это время в таблице данных 1 на рабочем листе.

7

Отцепите пружину и выдвиньте индикатор радиуса примерно на 1 см. Измерьте и запишите это новое значение радиуса в Таблицу данных 1.

8

Повторите шаги с 4 по 6 и введите значения в таблицу данных 1.

9

Повторите шаг 7 для еще трех позиций индикатора радиуса, всего 5 радиусов.

КПП 1:
Попросите своего технического специалиста проверить значения в таблице данных 1, прежде чем продолжить.

10

Рассчитайте и запишите период вращения

T

в Таблицу данных 2.

11

Рассчитайте и запишите частоту вращения

f

в Таблицу данных 2.

12

Рассчитайте и запишите центростремительное ускорение в таблице данных 2.

13

Рассчитайте центростремительную силу.

14

Вычислите процентную разницу между экспериментальным значением центростремительной силы и силой подвешенного груза,

м h г

, и запишите значения в Таблицу данных 2. См. Приложение B.

Примечание : Если ваша процентная разница превышает 15% для одного испытания, вы, , должны повторить это испытание.

КПП 2:
Попросите своего технического специалиста проверить значения и расчеты в таблице данных 2.

Процедура B: Участок

м h g

по сравнению с

a c

15

Используя Excel, постройте график

m h g

в сравнении с

a c

. См. Приложение G.

16

Используйте опцию линии тренда в Excel, чтобы найти наклон.См. Приложение H.

17

По наклону определяют величину вращающейся массы.

18

Сравните значение вращающейся массы, полученное из уклона, с измеренным значением, вычислив процентную разницу.

КПП 3:
Попросите своего TA проверить ваш график и расчеты.

Авторские права © 2010 Advanced Instructional Systems, Inc.и Государственный университет Северной Каролины. | Кредиты

.

Добавить комментарий

Ваш адрес email не будет опубликован. Обязательные поля помечены *